SlideShare a Scribd company logo
1 of 8
Download to read offline
500 | july 2014 | volume 44 | number 7 | journal of orthopaedic & sports physical therapy
[ resident’s case problem ]
L
ow back pain is a common condition routinely managed by
physical therapists and accounts for up to 53% of all client
visits.1
It is also the fifth most common reason for all physician
visits in the United States,19
with direct health care costs related
to the management of low back pain in the United States estimated
to be more than $85 billion annually.30
Unfortunately, despite increasing medi-
cal expenditures, the prevalence of chron-
ic, disabling low back pain continues to
increase.30
Low back pain is most commonly
caused by mechanical dysfunction, which
can be defined as symptoms that are re-
lated to the musculoskeletal system and
vary with movement.40
However, serious
nonmechanical spinal disorders (eg, neo-
plasm, infection, inflammatory arthritis,
fracture) or visceral disease (eg, gastroin-
testinal, genitourinary, vascular) may be
the cause of low back pain in a small per-
centage of patients.22
Though mechanical
dysfunction accounts for approximately
97% of low back pain cases, nonmechani-
cal spinal disorders and visceral disease
account for approximately 1% and 2% of
low back pain cases, respectively.22
One type of visceral disease that may
cause low back pain is an abdominal aor-
tic aneurysm (AAA), which is an abnor-
mal dilation in a weakened or diseased
arterial wall.15
By convention, an infra-
renal aorta that is 3 cm in diameter or
larger is considered aneurysmal.12,26
The
diameter of the infrarenal aorta is the
strongest known predictor of risk for rup-
ture,2
with the risk substantially increas-
ing with infrarenal aorta diameters of 5.0
cm or greater.14,18
The etiology of an AAA is not com-
pletely understood, but atherosclerosis,
degeneration, and chronic inflammation
have been identified as contributing to
its development.39
Major risk factors for
an AAA include hypertension, an age of
greater than 60 years, male sex, past his-
tory of smoking, atherosclerosis, coronary
artery disease, family history of AAA, and
use of statins (TABLE 1).2,7,12,13,26
Factors that
are negatively associated with AAA include
female sex, black race, and diabetes.2,7,13,24,25
The purpose of this resident’s case
problem was (1) to describe the clini-
cal reasoning and decision making in a
patient referred with a diagnosis of me-
chanical low back pain who had an AAA
TTSTUDY DESIGN: Resident’s case problem.
TTBACKGROUND: The purpose of this report was
to describe (1) the clinical reasoning that led a
clinician to identify an abdominal aortic aneurysm
(AAA) in a patient with low back pain requiring
immediate medical referral, and (2) an evidence-
based approach to clinical evaluation of patients
with suspected AAA.
TTDIAGNOSIS: The patient was unable to identify
a specific mechanism of injury for his low back
pain, lacked aggravating/easing factors for his
symptoms, and complained of night pain and
an inability to ease his symptoms with position
changes. While the patient’s symptoms remained
unchanged during physical examination of the lum-
bar spine and hip, abdominal palpation revealed
a strong, nontender pulsation over the midline of
the upper and lower abdominal quadrants. Due to
concern for an AAA, the patient was immediately
referred to his physician. Subsequent computed
tomography imaging revealed a prominent AAA,
which measured up to 5.5 cm in greatest dimen-
sion and extended from below the renal arteries to
the bifurcation of the iliac arteries. The patient ini-
tially deferred surgical intervention but eventually
consented 6 months later, after repeat computed
tomography imaging revealed that the AAA had
progressed to 6.7 cm in greatest dimension.
TTDISCUSSION: It is essential for physical
therapists to be familiar with a diagnostic pathway
to help identify AAA in patients presenting with
apparent musculoskeletal complaints. Knowledge
of the risk factors for AAA, understanding how to
screen for nonmusculoskeletal symptoms, and a
basic competence in abdominal palpation and how
to interpret findings will help with the clinician’s
clinical decision making.
TTLEVEL OF EVIDENCE: Differential diagnosis,
level 4. J Orthop Sports Phys Ther 2014;44(7):500-
507. Epub 25 April 2014. doi:10.2519/
jospt.2014.4935
TTKEY WORDS: abdomen, aorta, clinical
reasoning, palpation
1
Department of Physical Therapy, Charleston Air Force Base, Charleston, SC. 2
Physical Therapy Department, Daemen College, Amherst, NY. 3
Department of Physical Therapy,
Fort McNair, Washington, DC. The opinions or assertions contained herein are the private views of the authors and are not to be construed as official or as reflecting the views of
the US Air Force or the Department of Defense. The authors certify that they have no affiliations with or financial involvement in any organization or entity with a direct financial
interest in the subject matter or materials discussed in the article. Address correspondence to Dr Joshua J. Van Wyngaarden, 1311 Hitchcock Avenue, Charleston AFB, SC 29404.
E-mail: josh.vanwyngaarden@gmail.com t Copyright ©2014 Journal of Orthopaedic & Sports Physical Therapy®
JOSHUA J. VAN WYNGAARDEN, PT, DPT1
• MICHAEL D. ROSS, PT, DHSc2
• BENJAMIN R. HANDO, PT, DSc3
Abdominal Aortic Aneurysm
in a Patient With Low Back Pain
44-07 Van Wyngaarden.indd 500 6/17/2014 7:37:25 PM
JournalofOrthopaedic&SportsPhysicalTherapy®
Downloadedfromwww.jospt.orgatonAugust12,2014.Forpersonaluseonly.Nootheruseswithoutpermission.
Copyright©2014JournalofOrthopaedic&SportsPhysicalTherapy®.Allrightsreserved.
journal of orthopaedic & sports physical therapy | volume 44 | number 7 | july 2014 | 501
requiring immediate medical referral,
and (2) to describe an evidence-based ap-
proach to clinical evaluation of patients
with suspected AAA.
DIAGNOSIS
Chief Complaint
T
he patient was a 58-year-old
Caucasian man referred to a physi-
cal therapist with a primary com-
plaint of low back pain and a secondary
complaint of abdominal pain. The pa-
tient was employed as a government
contractor, and his job consisted primar-
ily of desk work. The patient performed
30 minutes of cardiovascular exercise
(elliptical machine) 7 days per week and
weight training 6 days per week. He had
continued this exercise schedule despite
his low back and abdominal pain.
Symptom Location, Description,
and Behavior
The patient’s low back pain was located
centrally at the L3 through L5 spinal
levels. He characterized the pain as a
constant, deep, dull ache that varied in
intensity, with intermittent throbbing
sensations (FIGURE 1). During the physical
therapist’s initial encounter, the patient
rated his current resting pain as 3 on a
verbal numeric pain rating scale ranging
from 0 to 10, with 0 as no pain and 10
as the worst pain imaginable. Using the
same pain rating scale, the patient rated
his pain since the pain first presented as
3 at its lowest level and 7 at its typical
maximum level.
The patient also reported abdominal
pain that was centrally located, start-
ing in the epigastric region and extend-
ing inferiorly to the superior margin of
the hypogastric region. He described his
abdominal pain as a bloated feeling that
progressed to a sharp, intense stabbing
pain when aggravated. He currently rated
his abdominal pain as 1; he also stated
that 1 was the minimum pain he could
feel, and the typical maximum pain was
4. The patient expressed that the low
back pain and abdominal pain seemed to
be related and increased proportionally;
that is, when the low back pain increased,
the abdominal pain increased as well.
Symptom Behavior
The patient was unable to describe any
specific aggravating factors for his low
back or abdominal pain. More specifical-
ly, he denied any difficulty or increase in
symptoms with aerobic exercise, climb-
ing stairs, lumbar flexion/extension ac-
tivities, or resistance training. He also
denied an increase in symptoms with
coughing, sneezing, and taking a deep
breath. Taking naproxen was the only
easing factor the patient identified, and it
would reportedly lessen both his low back
and abdominal pain temporarily.
He had difficulty falling asleep at
night, because the pain in his lower back
would increase and be followed by an
increase in abdominal pain. Once he fell
asleep, he would typically wake at least
once each night because of pain in his low
back and abdomen. Position changes did
not increase or decrease his symptoms.
TABLE 1
Risk Factors Associated
With Abdominal Aortic Aneurysm
and Their Corresponding Odds Ratios7,13
Risk Factor Odds Ratio*
Age 1.71 (1.61, 1.82) increase for every 7-y increase
Male sex†
2.66 (1.69, 4.20)
Hypertension†
1.44 (1.27, 1.63)
Smoking history
Past history†
1.5-2.4 (0.54, 6.75)
Current smoker 6.19-13.72 (2.86, 30.78)
Atherosclerosis†
Not reported in literature
Family history 1.94 (1.63, 2.32)
Coronary artery disease 1.52 (1.37, 1.68)
Use of statins 3.77 (1.45, 9.81)
*Values in parentheses are 95% confidence interval.
†
Indicates the risk factors present in this patient.
FIGURE 1. Location and description of patient’s symptoms at the time of the initial physical therapy visit.
Checkmarks denote symptom-free areas.
44-07 Van Wyngaarden.indd 501 6/17/2014 7:37:25 PM
JournalofOrthopaedic&SportsPhysicalTherapy®
Downloadedfromwww.jospt.orgatonAugust12,2014.Forpersonaluseonly.Nootheruseswithoutpermission.
Copyright©2014JournalofOrthopaedic&SportsPhysicalTherapy®.Allrightsreserved.
502 | july 2014 | volume 44 | number 7 | journal of orthopaedic & sports physical therapy
[ resident’s case problem ]
To resume sleeping, he would typically
have to lie in bed and wait 45 minutes
to 1 hour until he fell asleep, or take one
500-mg naproxen.
History of Current Condition
The patient reported that his pain began
8 days prior to his initial evaluation by
the physical therapist. He denied a spe-
cific mechanism of injury or trauma that
preceded his symptoms. His symptoms
came on suddenly while sitting in his ho-
tel room when he was away on a business
trip. He described his initial symptoms as
a “sudden, severe pain” in his lower back
and abdomen that forced him to lie su-
pine for 30 minutes. Over the course of
the next 8 days, the pain in his low back
and abdomen slowly improved. The pa-
tient denied a personal past history of
low back or abdominal pain prior to this
incident.
One day prior to being seen by the
physical therapist, the patient was evalu-
ated by his primary care physician and
diagnosed with mechanical low back pain
and abdominal pain that was related to
the low back pain, both of which were
deemed musculoskeletal in nature. Prior
to referral to the physical therapist, plain-
film radiographs were completed, which
revealed mild degenerative changes of
the L3 through S1 spinal levels. The phy-
sician subsequently prescribed naproxen
(500 mg), to be taken twice daily for the
next 10 to 14 days, and a physical therapy
referral.
Past Medical History
The patient had a history of hyperten-
sion and atherosclerosis, for which he
was currently taking amlodipine (5 mg),
vytorin (10 mg), hyzaar (12.5 mg), and
aspirin (81 mg) once daily. He indicated
that he had smoked for 2 years in the
early 1970s, amounting to 1 pack-year
total, but has not used tobacco products
since. He denied numbness and tingling
in his upper and lower extremities, recent
infection, fatigue, weight changes, fever,
malaise, saddle anesthesia, pulsation in
his abdomen, and pain with the Valsalva
maneuver or coughing. Heartburn, in-
digestion, changes in appetite, difficulty
swallowing, and changes in bowel and
bladder habit were also all denied. He
denied a family history of hypertension,
diabetes mellitus, cancer, coronary artery
disease, and AAA.
Clinical Impression Post–History Intake
The patient had a primary complaint of
low back pain and a secondary complaint
of abdominal pain that appeared to be re-
lated. When considering the differential
diagnosis, initial hypotheses were con-
sidered in 3 categories: mechanical (eg,
musculoskeletal condition), nonmechan-
ical (eg, neoplasm, infection, inflamma-
tory arthritis, fracture), and visceral (eg,
gastrointestinal, genitourinary, or vascu-
lar). This approach to differential diag-
nosis allowed the clinician to determine
whether the patient’s condition was ap-
propriate for physical therapist manage-
ment or an additional physician consult
was required.3
Mechanical dysfunction accounts for
approximately 97% of low back pain cas-
es, suggesting a high probability that the
vast majority of patients with low back
pain will have pain that is mechanical
in nature. Lumbar dysfunction may also
cause abdominal pain.10,23
From the pa-
tient history, however, the physical ther-
apist identified components suggesting
that the patient’s symptoms might not be
mechanical in origin, such as the patient’s
inability to identify a specific mechanism
of injury to the sudden onset of symp-
toms, the inability to identify any aggra-
vating or easing factors for his symptoms,
and night pain with an inability to ease
his symptoms with changes in position.
Additionally, Sparkes et al37
developed 2
specific clusters of questions to be used in
identifying patients with abdominal pain
that is musculoskeletal in origin (TABLE
2). These question clusters have high
specificity (96%), suggesting that this
tool is most useful for ruling in a muscu-
loskeletal cause of abdominal pain. The
patient described in this report could not
answer “yes” to either of the first 2 ques-
tions of cluster 1, suggesting that he was
negative for this question cluster, which
decreased the likelihood for a musculo-
skeletal origin of the patient’s abdominal
symptoms.37
The sensitivity (0.67) and
negative likelihood ratio (LR) (0.39) as-
sociated with the cluster are relatively
weak, suggesting that a musculoskeletal
cause of the patient’s symptoms was still
high. However, when considered in the
context of the other historical findings,
the fact that cluster 1 was negative gave
the clinician added suspicion that the
symptoms could be related to nonmus-
culoskeletal pathology.
There were also components of the
TABLE 2
Musculoskeletal Abdominal
Pain Questionnaire37
Cluster 1*
1. Does coughing, sneezing, or taking a deep breath make your pain feel worse? (yes)
2. Do activities such as bending, sitting, lifting, twisting, or turning over in bed make your pain feel worse? (yes)
3. Has there been any change in your bowel habit since the start of your symptoms? (no)
Cluster 2†
1. Does eating certain foods make your pain feel worse? (no)
2. Has your weight changed since your symptoms started? (no)
*A patient answering “yes” to either of the first 2 and “no” to the third question of cluster 1 suggests a
high probability that abdominal pain is musculoskeletal in origin (sensitivity, 0.67; specificity, 0.84;
positive likelihood ratio, 4.2; negative likelihood ratio, 0.39).
†
If the patient answers “no” to both questions in cluster 2, in addition to corresponding responses to
the questions in cluster 1, there is an even greater probability that abdominal pain is musculoskeletal
in origin (sensitivity, 0.67; specificity, 0.96; positive likelihood ratio, 16.8; negative likelihood ratio,
0.34).
44-07 Van Wyngaarden.indd 502 6/17/2014 7:37:26 PM
JournalofOrthopaedicSportsPhysicalTherapy®
Downloadedfromwww.jospt.orgatonAugust12,2014.Forpersonaluseonly.Nootheruseswithoutpermission.
Copyright©2014JournalofOrthopaedicSportsPhysicalTherapy®.Allrightsreserved.
journal of orthopaedic  sports physical therapy | volume 44 | number 7 | july 2014 | 503
patient history that the physical therapist
identified and assisted in determining if
the patient’s symptoms were nonmechan-
ical in origin. The incidence of cancer as a
cause of low back pain in a primary care
setting is 0.66%, or approximately 6 to
7 cases for every 1000 patients with low
back pain.8
Therefore, while the probabil-
ity of cancer as a cause of low back pain
is low, evidence-based screening strate-
gies suggest that cancer may be ruled
out with 100% sensitivity as a cause of
low back pain if there is no previous his-
tory of cancer, the patient is improving
with conservative medical treatment, the
patient is younger than 50 years of age,
and there is no unexplained weight loss.8
These 4 clinical findings also had high
positive LRs when assessing the accu-
racy of clinical findings for detecting the
presence of cancer resulting in low back
pain.8
The patient described in this report
had 3 of the 4 findings that allow cancer
to be ruled out with 100% sensitivity.
Although cancer could not be ruled out
with 100% sensitivity, it is a rare cause
of low back pain, and the patient did
not demonstrate any of the strong pre-
dictive findings for cancer as a cause of
low back pain.8
In addition, Henschke et
al20
conducted a Cochrane review to as-
sess the diagnostic accuracy of red flags
in screening for spinal malignancy in
patients with low back pain. This review
found a previous history of malignancy
to be a clinically meaningful finding for
detecting cancer. Given these factors, in
addition to the relatively benign findings
on plain-film radiographs, it was believed
that cancer was an unlikely cause of low
back pain. Given that the patient did not
report a fever, fatigue, or malaise, it was
believed that an infection was also un-
likely.3,22,31,41
Additionally, given the lack
of constitutional symptoms and no re-
ports of morning stiffness, the likelihood
of inflammatory arthritis as a cause of the
patient’s low back and abdominal pain
appeared to be low as well.17,22,31,34,35
Final-
ly, it was highly unlikely that the patient
had a spinal fracture. Vertebral fractures
are the most common serious pathology
associated with low back pain.21
Red-flag
questions that are useful in identifying
spinal fracture include significant trauma
(positive LR, 10.03; negative LR, 0.77),
older age (positive LR, 11.19; negative
LR, 0.52), and prolonged corticosteroid
use (positive LR, 48.50; negative LR,
0.75).21,42
Henschke et al21
created a diag-
nostic rule to identify vertebral fracture
using the following variables: female sex,
age greater than 70 years, significant
trauma (major in young patients, minor
in elderly patients), and prolonged use of
corticosteroids. When at least 1 of these
variables was positive, the positive and
negative LRs were 1.8 and 0.24, respec-
tively.21
With 2 positive features, the posi-
tive and negative LRs increased to 15.5
and 0.39, respectively; 3 or more positive
variables yielded a positive LR of 218.3
and a negative LR of 0.62.21
In addition to
the absence of evidence of spinal fracture
on plain-film radiographs, none of these
variables were present in this patient,
further decreasing the likelihood that a
vertebral fracture was causing low back
pain.
While mechanical dysfunction of the
lumbar spine accounts for the vast major-
ity of symptoms experienced by patients
with low back pain, the majority of pa-
tients with abdominal pain usually have a
visceral cause, and in all patients with ab-
dominal pain, visceral pathology should
be ruled out.37
Furthermore, visceral dis-
ease is a more frequent cause of low back
pain compared to other nonmechanical
spinal disorders.22
With regard to visceral
disorders, initial screening questions for
the gastrointestinal (eg, patient denied
heartburn, nausea, indigestion, changes
in appetite, difficulty swallowing, consti-
pation, and changes in frequency of bowel
movements) and genitourinary (patient
denied pain with urination or changes in
urinary frequency) systems were nega-
tive.3
Although each of these findings
does not necessarily rule out the presence
of a gastrointestinal or genitourinary dis-
order, the cluster of negative findings did
suggest that the patient’s symptoms were
most likely not related to these systems.
Considering vascular pathology, an AAA
is a rare cause of low back pain, and in
many cases patients experience no symp-
toms at all.27
However, the patient in this
case indicated that he had 4 of the 8 risk
factors associated with increased preva-
lence of AAA (TABLE 1).2,7,12,13,26
Physical Examination
The patient’s height was 188 cm, and he
weighed 90.7 kg the day of the examina-
tion (body mass index, 25.7 kg/m2
). Vi-
sual observation revealed a nonantalgic
gait. A postural examination revealed a
forward head and rounded shoulders,
with a decreased lumbar lordosis. The
iliac crests, anterior superior iliac spine,
posterior superior iliac spine, and greater
trochanters were level and in the same
horizontal plane bilaterally.
Lumbar active range of motion and
associated symptom responses were as-
sessed with the patient in the standing
position. The low back and abdominal
pain remained unchanged during active
range-of-motion testing, and all motions
were within normal limits. Because the
patient’s symptoms remained unchanged
with active range-of-motion assessment,
overpressure at the end range of active
range of motion was applied to lumbar
flexion, extension, and bilateral sidebend-
ing. The low back and abdominal pain
also remained unchanged during over-
pressure assessment for each movement.
Seated hip active range of motion for
flexion, internal rotation, and external ro-
tation was within normal limits and pain
free bilaterally. Passive range of motion
with overpressure was then applied for
hip flexion, internal rotation, and exter-
nal rotation bilaterally, with the patient
lying supine. Passive left hip flexion with
overpressure slightly increased the pa-
tient’s low back pain to 4 on the numeric
pain rating scale, but the pain immedi-
ately returned to the patient’s baseline
pain level of 3 when his hip was taken
out of the position of left hip flexion with
overpressure. The patient did not note a
change in his abdominal pain with hip
range-of-motion assessment.
44-07 Van Wyngaarden.indd 503 6/17/2014 7:37:26 PM
JournalofOrthopaedicSportsPhysicalTherapy®
Downloadedfromwww.jospt.orgatonAugust12,2014.Forpersonaluseonly.Nootheruseswithoutpermission.
Copyright©2014JournalofOrthopaedicSportsPhysicalTherapy®.Allrightsreserved.
504 | july 2014 | volume 44 | number 7 | journal of orthopaedic  sports physical therapy
[ resident’s case problem ]
With the patient positioned prone,
spring testing through posterior-to-an-
terior pressures over the spinous pro-
cesses of T10 through L4 suggested mild
hypomobility but no increase in the pa-
tient’s symptoms. However, the patient
did report a slight increase in low back
pain with spring testing at the L5 level,
and there was hypomobility noted at this
level as well. A sacral spring test was then
performed, which did not change the pa-
tient’s symptoms. The patient denied any
change in his abdominal pain with spring
testing from T10 through S1.
The patient was then positioned su-
pine, and an abdominal assessment was
performed. Visual inspection of the pa-
tient’s abdomen did not reveal any skin
abnormalities, abdominal masses, or
abnormal movement of the abdominal
wall. Systematic palpation in each of the
4 abdominal quadrants did not provoke
an increase in the patient’s symptoms.
However, the clinician immediately no-
ticed the presence of a very strong ab-
dominal pulsation that lateralized from
the midline bilaterally and was measured
at approximately 5 cm in width. The pul-
sation was palpable from the epigastric
to the hypogastric regions. Following
this finding, the clinician elected to im-
mediately stop the objective examination
and contacted the patient’s primary care
provider to discuss the findings from the
evaluation.
Clinical Impression Post–Physical
Examination
The patient’s low back and abdominal
pain was not reproduced with any of the
physical examination tests and measures
performed, with the exception of a mild,
temporary increase in low back pain
with overpressure into hip flexion and
L5 palpation. In the majority of patients
with nonspecific mechanical low back
pain, pain is reproduced with lumbar
range-of-motion testing and/or lumbar
palpation.4,5
Therefore, the absence of
symptom reproduction during the physi-
cal examination further elevated concern
for the presence of visceral pathology,
which led to a thorough palpation of the
abdomen. During abdominal palpation, a
very strong, nontender abdominal pulsa-
tion at least 5 cm in width was detected,
which was concerning for AAA.3,31
Diagnosis
After discussing the case, the referring
physician immediately ordered an ultra-
sound imaging study of the abdomen,
which demonstrated dilation of the dis-
tal abdominal aorta up to 5.5 cm wide
(FIGURE 2). The physician subsequently
ordered a computed tomography scan
confirming the presence of a prominent
AAA measuring 5.5 cm in greatest di-
mension and an 80° angulation (FIGURE
3). The patient was immediately referred
to a vascular surgeon. However, after a
thorough discussion with the vascular
surgeon, the patient decided to forgo
immediate surgical intervention, believ-
ing that the risks incurred from surgery
outweighed the potential benefit. Given
this decision, the patient was instructed
to return biannually for repeat computed
tomography scanning to determine if the
AAA was progressing in size. The patient
reported that his primary complaint of
low back pain and secondary complaint
of abdominal pain had completely sub-
sided approximately 3 weeks after the
initial onset of symptoms.
Six months following the initial di-
agnosis of the AAA, repeat computed
tomography scanning revealed that the
FIGURE 3. Initial computed tomography image,
which demonstrated a prominent abdominal aortic
aneurysm, which measured up to 5.6 cm in greatest
dimension (line) and extended from below the renal
arteries to the bifurcation of the iliac arteries. The
aorta also buckled to the right, with approximately
80° of angulation present 1.3 cm distal to the left
renal artery (arrow).
FIGURE 2. Initial ultrasound image demonstrating a 5.5-cm dilation of the distal abdominal aorta (region between
the plus signs).
44-07 Van Wyngaarden.indd 504 6/17/2014 7:37:27 PM
JournalofOrthopaedicSportsPhysicalTherapy®
Downloadedfromwww.jospt.orgatonAugust12,2014.Forpersonaluseonly.Nootheruseswithoutpermission.
Copyright©2014JournalofOrthopaedicSportsPhysicalTherapy®.Allrightsreserved.
journal of orthopaedic  sports physical therapy | volume 44 | number 7 | july 2014 | 505
aneurysm had increased by 1.2 to 6.7
cm in greatest dimension and to 90° of
angulation (FIGURE 4). Interestingly, the
patient reported that he did not have any
further episodes of low back or abdomi-
nal pain since the symptoms resolved ap-
proximately 5 months earlier, despite the
aneurysm’s progression. The average rate
of expansion of the typical AAA is from
0.3 to 0.4 cm per year.14,18,29
Patients who
have AAA expansion rates of 0.5 cm in a
12-month time frame are considered can-
didates for elective surgical repair.16
Due
to the rapid progression of the AAA, the
patient accepted the risks of an open ab-
dominal repair. The patient requested an
endovascular repair, but the increasing
angulation of the aorta and the weaken-
ing aortic wall made an open repair the
only viable option. TABLE 3 provides rela-
tive dates of the patient’s medical visits
and procedures.
Following surgery, the patient re-
turned to walking 30 minutes to 1 hour
daily. At 6 months following surgery, the
patient was allowed to initiate a low-
weight, high-repetition weight-training
routine. At 9 months following surgery,
the patient did not report any instances
of low back or abdominal pain since the
repair.
DISCUSSION
I
dentifying individuals with AAA is
a challenging task for any clinician.
There are no established algorithms
or clinical prediction rules to determine
with reasonable certainty whether an
AAA is present in a patient. Additionally,
many AAAs are completely asymptom-
atic, which further complicates the clini-
cal picture.27
However, there is evidence
in the peer-reviewed literature that can
inform clinicians in their clinical reason-
ing and thought process. Knowledge of
the risk factors for AAA, systematic ap-
plication of screening techniques for non-
musculoskeletal symptoms, and a basic
competence in abdominal palpation and
how to interpret findings can help iden-
tify AAA in a patient presenting with ap-
parent musculoskeletal complaints.
Understanding the risk factors for
AAA can help identify patients who
should be referred for further testing.
The major risk factors and associated
odds ratios for AAA are listed in TABLE
1. There is discrepancy in the literature
as far as when patients should undergo
medical screening for AAA.6,12,28
In obese
male patients over the age of 60, Led-
erle et al28
recommended ultrasound
screening examinations regardless of
the physical examination findings. Flem-
ing et al12
conducted a systematic review
that looked at the benefits and harms of
population-based AAA screening. They
concluded that screening will reduce
age-related mortality in men between
the ages of 65 and 75 years.12
According
to Cosford and Leng,7
an age of greater
than 65 years, past history of smok-
ing, male sex, and family history are the
most significant AAA risk factors (TABLE
1).7,12
Therefore, if a male patient in his
sixth decade of life has 1 or more addi-
tional risk factors for AAA, a referral to
the primary care provider with a recom-
mendation for an ultrasound screening
examination may be indicated.
In patients with complaint of abdomi-
nal pain, the question cluster suggested
by Sparkes et al37
can serve as a valuable
clinical decision tool when determining
whether a patient has symptoms of mus-
culoskeletal origin (TABLE 2). Additional-
ly, the characteristics of abdominal pain
can provide insight into the structures
responsible for symptoms. Abdominal
pain of musculoskeletal origin may pre­
sent as a deep, sharp, localized, cramping
and aching pain.32
In comparison, pain
FIGURE 4. Computed tomography scan completed
6 months after the initial scan seen in FIGURE
3. This image demonstrated interval progression
of the abdominal aortic aneurysm to 6.7 cm in
greatest dimension (line). There was also significant
angulation at the neck of the aneurysm, measuring at
least 90° (arrow).
TABLE 3
Relative Timing of the Patient’s
Medical Visits or Procedures
Date Medical Visits or Procedures
Day 1 • Initial physical therapy visit for a primary complaint of low back pain and a secondary complaint of
abdominal pain
• Physical therapist informed the patient and the referring provider of the concern of an abdominal aortic
aneurysm
• Ultrasound imaging and computed tomography scan completed, which confirmed the presence of an
abdominal aortic aneurysm
• Initial vascular surgery visit
Day 17 • Second vascular surgery visit
Day 31 • Third vascular surgery visit
• Decision made to forgo immediate surgical intervention of abdominal aortic aneurysm
• Primary complaint of low back pain and secondary complaint of abdominal pain had completely
subsided
• Patient instructed to complete repeat computed tomography scan in 6 mo
Day 204 • Fourth vascular surgery visit
• Repeat computed tomography scan results evaluated, which demonstrated interval progression of
abdominal aortic aneurysm
Day 302 • Open vascular repair of abdominal aortic aneurysm
44-07 Van Wyngaarden.indd 505 6/17/2014 7:37:28 PM
JournalofOrthopaedicSportsPhysicalTherapy®
Downloadedfromwww.jospt.orgatonAugust12,2014.Forpersonaluseonly.Nootheruseswithoutpermission.
Copyright©2014JournalofOrthopaedicSportsPhysicalTherapy®.Allrightsreserved.
506 | july 2014 | volume 44 | number 7 | journal of orthopaedic  sports physical therapy
[ resident’s case problem ]
REFERENCES
	1. Battie MC, Cherkin DC, Dunn R, Ciol MA, Wheel-
er KJ. Managing low back pain: attitudes and
treatment preferences of physical therapists.
Phys Ther. 1994;74:219-226.
	2. Blanchard JF, Armenian HK, Friesen PP. Risk
factors for abdominal aortic aneurysm: re-
sults of a case-control study. Am J Epidemiol.
2000;151:575-583.
	3. Boissonnault WG. Primary Care for the Physical
Therapist: Examination and Triage. 2nd ed. St
Louis, MO: Elsevier/Saunders; 2011.
	4. Boissonnault WG, Bass C. Pathological origins
of trunk and neck pain: part I—pelvic and
abdominal visceral disorders. J Orthop Sports
Phys Ther. 1990;12:192-207. http://dx.doi.
org/10.2519/jospt.1990.12.5.192
	5. Boissonnault WG, Bass C. Pathological origins
of trunk and neck pain: part II—disorders of the
cardiovascular and pulmonary systems. J Or-
thop Sports Phys Ther. 1990;12:208-215. http://
dx.doi.org/10.2519/jospt.1990.12.5.208
	6. Chervu A, Clagett GP, Valentine RJ, Myers SI,
Rossi PJ. Role of physical examination in detec-
tion of abdominal aortic aneurysms. Surgery.
1995;117:454-457.
	7. Cosford PA, Leng GC. Screening for ab-
dominal aortic aneurysm. Cochrane Database
Syst Rev. 2007:CD002945. http://dx.doi.
org/10.1002/14651858.CD002945.pub2
	8. Deyo RA, Diehl AK. Cancer as a cause of
back pain: frequency, clinical presentation,
and diagnostic strategies. J Gen Intern Med.
1988;3:230-238.
	9. Edwards JZ, Weiner SD. Chronic back pain
caused by an abdominal aortic aneurysm: case
report and review of the literature. Orthopedics.
2003;26:191-192.
	10. Feinstein B, Langton JN, Jameson RM, Schil-
ler F. Experiments on pain referred from
deep somatic tissues. J Bone Joint Surg Am.
1954;36-A:981-997.
	11. Fink HA, Lederle FA, Roth CS, Bowles CA, Nelson
DB, Haas MA. The accuracy of physical exami-
nation to detect abdominal aortic aneurysm.
Arch Intern Med. 2000;160:833-836.
	12. Fleming C, Whitlock EP, Beil TL, Lederle FA.
Screening for abdominal aortic aneurysm: a
best-evidence systematic review for the U.S.
Preventive Services Task Force. Ann Intern Med.
2005;142:203-211.
	13. Forsdahl SH, Singh K, Solberg S, Jacob-
sen BK. Risk factors for abdominal aortic
aneurysms: a 7-year prospective study:
the Tromsø Study, 1994-2001. Circulation.
2009;119:2202-2208. http://dx.doi.org/10.1161/
CIRCULATIONAHA.108.817619
	14. Glimåker H, Holmberg L, Elvin A, et al. Natural
history of patients with abdominal aortic aneu-
rysm. Eur J Vasc Surg. 1991;5:125-130.
	15. Goodman CC, Snyder TEK. Differential Diagnosis
in Physical Therapy. 3rd ed. Philadelphia, PA:
arising from visceral tissue is often de-
scribed as dull, aching, cramping, throb-
bing, burning, gnawing, or wave-like and
is often poorly localized.32
Movement typically alters symptoms
associated with musculoskeletal pain.4,5
In contrast, nonmusculoskeletal pain
can be characterized as pain that cannot
be provoked, alleviated, or eliminated by
movement or position changes.4,5,15
If a
patient is unable to identify specific ag-
gravating and easing factors and the cli-
nician is unable to reproduce symptoms
with provocative testing, then a nonmus-
culoskeletal origin of pain should be con-
sidered and the case discussed with the
referring physician.3,32
Abdominal palpation can play an im-
portant role in identifying patients with
AAA. However, the accuracy of palpation
depends on the width of the aneurysm
and the patient’s abdominal girth.11
Ab-
dominal palpation has only a moderate
overall ability in detecting smaller AAAs,
but does appear very useful in identify-
ing aneurysms large enough to warrant
elective repairs.11,28,29
The sensitivity of
palpation in patients with an abdominal
girth less than 100 cm and an AAA 5.0
cm or greater is 100%.11
The patient in
this case had a 95-cm waistline and had
an AAA greater than 5 cm, resulting in
a successful detection of the AAA during
the palpation examination. Palpation of
an AAA appears to be safe and has not
been reported to precipitate rupture.27
If
a palpable pulsation of 3.0 cm in width
or greater is noticed, physician referral is
indicated12
; however, it’s important to re-
member that aneurysms less than 5.0 cm
are not as easily identifiable with palpa-
tion. Therefore, sole reliance on abdomi-
nal palpation is not recommended.11,24
It is important to note that the clini-
cal examination plays a vital role in the
recognition of AAAs, but should not be
relied on to exclude their presence.29
If a referral to the primary care physi-
cian for suspected AAA is indicated, the
physician will likely order an abdominal
ultrasound, as this can confirm the pres-
ence or absence of an AAA (sensitivity,
0.97-1.0; specificity, 0.94-1.0; negative
LR, 0-0.025; positive LR, 10.8 to infin-
ity).28,33
Ultrasound screening is not only
highly accurate in detecting AAAs33
but
is also low cost and does not expose the
patient to radiation.6,22,28
If the presence
of an AAA is confirmed with ultrasound
imaging, a computed tomography scan
is the preferred imaging progression.16,38
While the sensitivity and specificity of
computed tomography imaging of the ab-
domen are lower than that of ultrasound
imaging (sensitivity, 0.79-0.90; specific-
ity, 0.77-0.91),36
it can detect additional
aneurysms, evaluate the surrounding
organs, and provide details on arterial-
wall characteristics (dissection, rupture,
or thickening).16
Computed tomogra-
phy does have one major disadvantage:
it exposes the patient to radiation.16,38
Magnetic resonance angiography yields
similar results to those of computed to-
mography without exposure to ionizing
radiation, but high cost and limited avail-
ability have prevented widespread adop-
tion of this modality.16,38
CONCLUSION
A
lthough the majority of indi-
viduals with AAA are asymp-
tomatic at the time of diagnosis,
some patients do present with a history
and clinical presentation suggestive of
AAA.9
This resident’s case problem de-
scribed the decision making and clinical
reasoning that led a clinician to cor-
rectly identify the presence of an AAA
in a patient referred from a primary care
provider with a diagnosis of mechanical
low back pain. Findings from the sub-
jective interview and physical examina-
tion led the clinician to suspect AAA as
an explanation for the patient’s com-
plaints. This suspicion was confirmed
with a series of imaging studies, which
led to expedited medical care and even-
tual surgical intervention. t
44-07 Van Wyngaarden.indd 506 6/17/2014 7:37:29 PM
JournalofOrthopaedicSportsPhysicalTherapy®
Downloadedfromwww.jospt.orgatonAugust12,2014.Forpersonaluseonly.Nootheruseswithoutpermission.
Copyright©2014JournalofOrthopaedicSportsPhysicalTherapy®.Allrightsreserved.
journal of orthopaedic  sports physical therapy | volume 44 | number 7 | july 2014 | 507
@ MORE INFORMATION
WWW.JOSPT.ORG
W.B. Saunders; 2000.
	16. Gorski Y, Ricotta JJ. Weighing risks in abdominal
aortic aneurysm. Best repaired in an elective,
not an emergency, procedure. Postgrad Med.
1999;106:69-70, 75-80.
	17. Gran JT. An epidemiological survey of the signs
and symptoms of ankylosing spondylitis. Clin
Rheumatol. 1985;4:161-169.
	18. Guirguis EM, Barber GG. The natural history
of abdominal aortic aneurysms. Am J Surg.
1991;162:481-483.
	19. Hart LG, Deyo RA, Cherkin DC. Physician of-
fice visits for low back pain. Frequency, clinical
evaluation, and treatment patterns from a
U.S. national survey. Spine (Phila Pa 1976).
1995;20:11-19.
	20. Henschke N, Maher CG, Ostelo RW, de Vet HC,
Macaskill P, Irwig L. Red flags to screen for
malignancy in patients with low-back pain. Co-
chrane Database Syst Rev. 2013;2:CD008686.
http://dx.doi.org/10.1002/14651858.CD008686.
pub2
	21. Henschke N, Maher CG, Refshauge KM, et al.
Prevalence of and screening for serious spinal
pathology in patients presenting to primary
care settings with acute low back pain. Arthritis
Rheum. 2009;60:3072-3080. http://dx.doi.
org/10.1002/art.24853
	22. Jarvik JG, Deyo RA. Diagnostic evaluation of
low back pain with emphasis on imaging. Ann
Intern Med. 2002;137:586-597. http://dx.doi.
org/10.7326/0003-4819-137-7-200210010-00010
	23. Kellgren JH. Referred pains from muscle. Br Med
J. 1938;1:325-327.
	24. Lederle FA, Johnson GR, Wilson SE, et al. The
aneurysm detection and management study
screening program: validation cohort and final
results. Aneurysm Detection and Management
Veterans Affairs Cooperative Study Investigators.
Arch Intern Med. 2000;160:1425-1430.
	25. Lederle FA, Johnson GR, Wilson SE, et al. Preva-
lence and associations of abdominal aortic an-
eurysm detected through screening. Aneurysm
Detection and Management (ADAM) Veterans
Affairs Cooperative Study Group. Ann Intern
Med. 1997;126:441-449.
	26. Lederle FA, Johnson GR, Wilson SE, et al. Rela-
tionship of age, gender, race, and body size to
infrarenal aortic diameter. The Aneurysm Detec-
tion and Management (ADAM) Veterans Affairs
Cooperative Study Investigators. J Vasc Surg.
1997;26:595-601.
	27. Lederle FA, Simel DL. The rational clinical exami-
nation. Does this patient have abdominal aortic
aneurysm? JAMA. 1999;281:77-82. http://dx.doi.
org/10.1001/jama.281.1.77
	28. Lederle FA, Walker JM, Reinke DB. Selective
screening for abdominal aortic aneurysms with
physical examination and ultrasound. Arch In-
tern Med. 1988;148:1753-1756.
	29. Lynch RM. Ruptured abdominal aortic aneu-
rysm presenting as groin pain. Br J Gen Pract.
2002;52:320-321.
	30. Martin BI, Deyo RA, Mirza SK, et al. Expendi-
tures and health status among adults with back
and neck problems. JAMA. 2008;299:656-664.
http://dx.doi.org/10.1001/jama.299.6.656
	31. Mechelli F, Preboski Z, Boissonnault WG. Dif-
ferential diagnosis of a patient referred to
physical therapy with low back pain: abdominal
aortic aneurysm. J Orthop Sports Phys Ther.
2008;38:551-557. http://dx.doi.org/10.2519/
jospt.2008.2719
	32. Rodeghero JR, Denninger TR, Ross MD. Abdomi-
nal pain in physical therapy practice: 3 patient
cases. J Orthop Sports Phys Ther. 2013;43:44-
53. http://dx.doi.org/10.2519/jospt.2013.4408
	33. Rubano E, Mehta N, Caputo W, Paladino L, Sin-
ert R. Systematic review: emergency department
bedside ultrasonography for diagnosing sus-
pected abdominal aortic aneurysm. Acad Emerg
Med. 2013;20:128-138. http://dx.doi.org/10.1111/
acem.12080
	34. Rudwaleit M, Metter A, Listing J, Sieper J, Braun
J. Inflammatory back pain in ankylosing spon-
dylitis: a reassessment of the clinical history for
application as classification and diagnostic cri-
teria. Arthritis Rheum. 2006;54:569-578. http://
dx.doi.org/10.1002/art.21619
	35. Sieper J, van der Heijde D, Landewé R, et al.
New criteria for inflammatory back pain in
patients with chronic back pain: a real patient
exercise by experts from the Assessment of
SpondyloArthritis international Society (ASAS).
Ann Rheum Dis. 2009;68:784-788. http://dx.doi.
org/10.1136/ard.2008.101501
	36. Silverstein MD, Pitts SR, Chaikof EL, Ballard
DJ. Abdominal aortic aneurysm (AAA): cost-
effectiveness of screening, surveillance of
intermediate-sized AAA, and management of
symptomatic AAA. Proc (Bayl Univ Med Cent).
2005;18:345-367.
	37. Sparkes V, Prevost AT, Hunter JO. Derivation
and identification of questions that act as
predictors of abdominal pain of musculo-
skeletal origin. Eur J Gastroenterol Hepatol.
2003;15:1021-1027. http://dx.doi.org/10.1097/01.
meg.0000059173.46867.0c
	38. Tang T, Boyle JR, Dixon AK, Varty K. Inflamma-
tory abdominal aortic aneurysms. Eur J Vasc
Endovasc Surg. 2005;29:353-362. http://dx.doi.
org/10.1016/j.ejvs.2004.12.009
	39. Thompson RW. Basic science of abdominal aor-
tic aneurysms: emerging therapeutic strategies
for an unresolved clinical problem. Curr Opin
Cardiol. 1996;11:504-518.
	40. Waddell G. The Back Pain Revolution. New York,
NY: Churchill Livingstone; 1998.
	41. Waldvogel FA, Papageorgiou PS. Osteo-
myelitis: the past decade. N Engl J Med.
1980;303:360-370. http://dx.doi.org/10.1056/
NEJM198008143030703
	42. Williams CM, Henschke N, Maher CG, et al. Red
flags to screen for vertebral fracture in patients
presenting with low-back pain. Cochrane Data-
base Syst Rev. 2013;1:CD008643. http://dx.doi.
org/10.1002/14651858.CD008643.pub2
NOTIFY JOSPT of Changes in Address
Please remember to let JOSPT know about changes in your mailing address.
The US Postal Service typically will not forward second-class periodical
mail. Journals are destroyed, and the USPS charges JOSPT for sending
them to the wrong address. You may change your address online at
www.jospt.org. Visit Info Center for Readers, click Change of Address, and
complete the online form. We appreciate your assistance in keeping
JOSPT’s mailing list up to date.
44-07 Van Wyngaarden.indd 507 6/17/2014 7:37:30 PM
JournalofOrthopaedicSportsPhysicalTherapy®
Downloadedfromwww.jospt.orgatonAugust12,2014.Forpersonaluseonly.Nootheruseswithoutpermission.
Copyright©2014JournalofOrthopaedicSportsPhysicalTherapy®.Allrightsreserved.

More Related Content

What's hot

The Depth and Breadth of Pain
The Depth and Breadth of PainThe Depth and Breadth of Pain
The Depth and Breadth of Painasclepiuspdfs
 
Pain Education For Nurses: Elderly Patients
Pain Education For Nurses: Elderly Patients Pain Education For Nurses: Elderly Patients
Pain Education For Nurses: Elderly Patients Raymond Zakhari
 
assessment and physiotherapy management of pain in elderly
assessment and physiotherapy management of pain in elderly assessment and physiotherapy management of pain in elderly
assessment and physiotherapy management of pain in elderly sunil JMI
 
Meanings of cancer-related pain
Meanings of cancer-related painMeanings of cancer-related pain
Meanings of cancer-related painSimon van Rysewyk
 
Fibromyalgia: Fact or Fiction? A Multi-disciplinary Approach
Fibromyalgia: Fact or Fiction? A Multi-disciplinary ApproachFibromyalgia: Fact or Fiction? A Multi-disciplinary Approach
Fibromyalgia: Fact or Fiction? A Multi-disciplinary ApproachMedicineAndHealthUSA
 
Pain Management in the Elderly
Pain Management in the ElderlyPain Management in the Elderly
Pain Management in the ElderlyAde Wijaya
 
Medical shockwaves for chronic low back pain - a case series
Medical shockwaves for chronic low back pain - a case seriesMedical shockwaves for chronic low back pain - a case series
Medical shockwaves for chronic low back pain - a case seriesKenneth Craig
 
Evidence based management of osteoarthritis in primary care - Dr Jonathan Quicke
Evidence based management of osteoarthritis in primary care - Dr Jonathan QuickeEvidence based management of osteoarthritis in primary care - Dr Jonathan Quicke
Evidence based management of osteoarthritis in primary care - Dr Jonathan Quickepcsciences
 
Chikungunya
ChikungunyaChikungunya
Chikungunyaashokha
 
Delirium (Charmaine Berggreen)
Delirium (Charmaine Berggreen)Delirium (Charmaine Berggreen)
Delirium (Charmaine Berggreen)honorhealth
 
Edwards et al-2006 Catastrophizing
Edwards et al-2006 CatastrophizingEdwards et al-2006 Catastrophizing
Edwards et al-2006 CatastrophizingPaul Coelho, MD
 

What's hot (19)

The Depth and Breadth of Pain
The Depth and Breadth of PainThe Depth and Breadth of Pain
The Depth and Breadth of Pain
 
Pain Education For Nurses: Elderly Patients
Pain Education For Nurses: Elderly Patients Pain Education For Nurses: Elderly Patients
Pain Education For Nurses: Elderly Patients
 
assessment and physiotherapy management of pain in elderly
assessment and physiotherapy management of pain in elderly assessment and physiotherapy management of pain in elderly
assessment and physiotherapy management of pain in elderly
 
Meanings of cancer-related pain
Meanings of cancer-related painMeanings of cancer-related pain
Meanings of cancer-related pain
 
Cancer Pain
Cancer PainCancer Pain
Cancer Pain
 
Pain in the elderly
Pain in the elderlyPain in the elderly
Pain in the elderly
 
Fibromyalgia: Fact or Fiction? A Multi-disciplinary Approach
Fibromyalgia: Fact or Fiction? A Multi-disciplinary ApproachFibromyalgia: Fact or Fiction? A Multi-disciplinary Approach
Fibromyalgia: Fact or Fiction? A Multi-disciplinary Approach
 
Pain in elderly
Pain in elderlyPain in elderly
Pain in elderly
 
Jbja0790303640
Jbja0790303640Jbja0790303640
Jbja0790303640
 
Pain Management in the Elderly
Pain Management in the ElderlyPain Management in the Elderly
Pain Management in the Elderly
 
Cancer pain management
Cancer pain managementCancer pain management
Cancer pain management
 
Medical shockwaves for chronic low back pain - a case series
Medical shockwaves for chronic low back pain - a case seriesMedical shockwaves for chronic low back pain - a case series
Medical shockwaves for chronic low back pain - a case series
 
Problem representation teaching slides
Problem representation teaching slidesProblem representation teaching slides
Problem representation teaching slides
 
Evidence based management of osteoarthritis in primary care - Dr Jonathan Quicke
Evidence based management of osteoarthritis in primary care - Dr Jonathan QuickeEvidence based management of osteoarthritis in primary care - Dr Jonathan Quicke
Evidence based management of osteoarthritis in primary care - Dr Jonathan Quicke
 
Illness Script Case Teaching Guide
Illness Script Case Teaching Guide Illness Script Case Teaching Guide
Illness Script Case Teaching Guide
 
Chikungunya
ChikungunyaChikungunya
Chikungunya
 
Topic of the month.... Back pain
Topic of the month.... Back painTopic of the month.... Back pain
Topic of the month.... Back pain
 
Delirium (Charmaine Berggreen)
Delirium (Charmaine Berggreen)Delirium (Charmaine Berggreen)
Delirium (Charmaine Berggreen)
 
Edwards et al-2006 Catastrophizing
Edwards et al-2006 CatastrophizingEdwards et al-2006 Catastrophizing
Edwards et al-2006 Catastrophizing
 

Similar to Jospt.2014 abd aneurysm

00006250 201409000-00021
00006250 201409000-0002100006250 201409000-00021
00006250 201409000-00021Jonny Luna
 
learn to solve several cases in low back pain
learn to solve several cases in low back painlearn to solve several cases in low back pain
learn to solve several cases in low back painHERRY632019
 
Diagnosis and Management of Chronic pain associated with depression.pptx
Diagnosis and Management of Chronic pain associated with depression.pptxDiagnosis and Management of Chronic pain associated with depression.pptx
Diagnosis and Management of Chronic pain associated with depression.pptxssuser40df77
 
I need a response to this assignmentzero plgiarismthree refe.docx
I need a response to this assignmentzero plgiarismthree refe.docxI need a response to this assignmentzero plgiarismthree refe.docx
I need a response to this assignmentzero plgiarismthree refe.docxflorriezhamphrey3065
 
STUDY PROTOCOL Open AccessPain coping skills training for .docx
STUDY PROTOCOL Open AccessPain coping skills training for .docxSTUDY PROTOCOL Open AccessPain coping skills training for .docx
STUDY PROTOCOL Open AccessPain coping skills training for .docxdeanmtaylor1545
 
Running Head GASTROINTESTINAL TRACT1GASTROINTESTINAL TRACT3.docx
Running Head GASTROINTESTINAL TRACT1GASTROINTESTINAL TRACT3.docxRunning Head GASTROINTESTINAL TRACT1GASTROINTESTINAL TRACT3.docx
Running Head GASTROINTESTINAL TRACT1GASTROINTESTINAL TRACT3.docxjeanettehully
 
Fibromyalgia Over-Diagnosed 97% of the time
Fibromyalgia Over-Diagnosed 97% of the timeFibromyalgia Over-Diagnosed 97% of the time
Fibromyalgia Over-Diagnosed 97% of the timeNelson Hendler
 
Riu008001 0028
Riu008001 0028Riu008001 0028
Riu008001 0028Jonny Luna
 
gtg_41 CPP.pdf
gtg_41 CPP.pdfgtg_41 CPP.pdf
gtg_41 CPP.pdfAmer Raza
 
Evaluation of Chest Pain in the Ambulatory Setting
Evaluation of Chest Pain in the Ambulatory SettingEvaluation of Chest Pain in the Ambulatory Setting
Evaluation of Chest Pain in the Ambulatory SettingKerolus Shehata
 
Acg guideline acute_pancreatitis_september_2013
Acg guideline acute_pancreatitis_september_2013Acg guideline acute_pancreatitis_september_2013
Acg guideline acute_pancreatitis_september_2013iberzamz
 
Am j hosp palliat care 2004-davis-137-42
Am j hosp palliat care 2004-davis-137-42Am j hosp palliat care 2004-davis-137-42
Am j hosp palliat care 2004-davis-137-42Esposito Evelyn
 
Catastrophizing and pain in arthritis, FMS, & other rheumatic diseases.
Catastrophizing and pain in arthritis, FMS, & other rheumatic diseases.Catastrophizing and pain in arthritis, FMS, & other rheumatic diseases.
Catastrophizing and pain in arthritis, FMS, & other rheumatic diseases.Paul Coelho, MD
 
Edwards et al-2006-arthritis_care
Edwards et al-2006-arthritis_careEdwards et al-2006-arthritis_care
Edwards et al-2006-arthritis_carePaul Coelho, MD
 
AssignmentAnalyze the possible conditions from your colleague.docx
AssignmentAnalyze the possible conditions from your colleague.docxAssignmentAnalyze the possible conditions from your colleague.docx
AssignmentAnalyze the possible conditions from your colleague.docxedmondpburgess27164
 
Differential Diagnosis of Lower Back Pain
Differential Diagnosis of  Lower Back PainDifferential Diagnosis of  Lower Back Pain
Differential Diagnosis of Lower Back Painwestwriters
 
Lower Back Pain - Part 2
Lower Back Pain - Part 2Lower Back Pain - Part 2
Lower Back Pain - Part 2cpppaincenter
 
Assessing back pain in rheumatology
Assessing back pain in rheumatologyAssessing back pain in rheumatology
Assessing back pain in rheumatologyDiana Girnita
 
Lindsay ClaggettWeek 8 Discussion Post Back PainCOLLAPSEW.docx
Lindsay ClaggettWeek 8 Discussion Post Back PainCOLLAPSEW.docxLindsay ClaggettWeek 8 Discussion Post Back PainCOLLAPSEW.docx
Lindsay ClaggettWeek 8 Discussion Post Back PainCOLLAPSEW.docxjesssueann
 

Similar to Jospt.2014 abd aneurysm (20)

00006250 201409000-00021
00006250 201409000-0002100006250 201409000-00021
00006250 201409000-00021
 
learn to solve several cases in low back pain
learn to solve several cases in low back painlearn to solve several cases in low back pain
learn to solve several cases in low back pain
 
Diagnosis and Management of Chronic pain associated with depression.pptx
Diagnosis and Management of Chronic pain associated with depression.pptxDiagnosis and Management of Chronic pain associated with depression.pptx
Diagnosis and Management of Chronic pain associated with depression.pptx
 
Dolor lumbar inespecífico
Dolor lumbar inespecíficoDolor lumbar inespecífico
Dolor lumbar inespecífico
 
I need a response to this assignmentzero plgiarismthree refe.docx
I need a response to this assignmentzero plgiarismthree refe.docxI need a response to this assignmentzero plgiarismthree refe.docx
I need a response to this assignmentzero plgiarismthree refe.docx
 
STUDY PROTOCOL Open AccessPain coping skills training for .docx
STUDY PROTOCOL Open AccessPain coping skills training for .docxSTUDY PROTOCOL Open AccessPain coping skills training for .docx
STUDY PROTOCOL Open AccessPain coping skills training for .docx
 
Running Head GASTROINTESTINAL TRACT1GASTROINTESTINAL TRACT3.docx
Running Head GASTROINTESTINAL TRACT1GASTROINTESTINAL TRACT3.docxRunning Head GASTROINTESTINAL TRACT1GASTROINTESTINAL TRACT3.docx
Running Head GASTROINTESTINAL TRACT1GASTROINTESTINAL TRACT3.docx
 
Fibromyalgia Over-Diagnosed 97% of the time
Fibromyalgia Over-Diagnosed 97% of the timeFibromyalgia Over-Diagnosed 97% of the time
Fibromyalgia Over-Diagnosed 97% of the time
 
Riu008001 0028
Riu008001 0028Riu008001 0028
Riu008001 0028
 
gtg_41 CPP.pdf
gtg_41 CPP.pdfgtg_41 CPP.pdf
gtg_41 CPP.pdf
 
Evaluation of Chest Pain in the Ambulatory Setting
Evaluation of Chest Pain in the Ambulatory SettingEvaluation of Chest Pain in the Ambulatory Setting
Evaluation of Chest Pain in the Ambulatory Setting
 
Acg guideline acute_pancreatitis_september_2013
Acg guideline acute_pancreatitis_september_2013Acg guideline acute_pancreatitis_september_2013
Acg guideline acute_pancreatitis_september_2013
 
Am j hosp palliat care 2004-davis-137-42
Am j hosp palliat care 2004-davis-137-42Am j hosp palliat care 2004-davis-137-42
Am j hosp palliat care 2004-davis-137-42
 
Catastrophizing and pain in arthritis, FMS, & other rheumatic diseases.
Catastrophizing and pain in arthritis, FMS, & other rheumatic diseases.Catastrophizing and pain in arthritis, FMS, & other rheumatic diseases.
Catastrophizing and pain in arthritis, FMS, & other rheumatic diseases.
 
Edwards et al-2006-arthritis_care
Edwards et al-2006-arthritis_careEdwards et al-2006-arthritis_care
Edwards et al-2006-arthritis_care
 
AssignmentAnalyze the possible conditions from your colleague.docx
AssignmentAnalyze the possible conditions from your colleague.docxAssignmentAnalyze the possible conditions from your colleague.docx
AssignmentAnalyze the possible conditions from your colleague.docx
 
Differential Diagnosis of Lower Back Pain
Differential Diagnosis of  Lower Back PainDifferential Diagnosis of  Lower Back Pain
Differential Diagnosis of Lower Back Pain
 
Lower Back Pain - Part 2
Lower Back Pain - Part 2Lower Back Pain - Part 2
Lower Back Pain - Part 2
 
Assessing back pain in rheumatology
Assessing back pain in rheumatologyAssessing back pain in rheumatology
Assessing back pain in rheumatology
 
Lindsay ClaggettWeek 8 Discussion Post Back PainCOLLAPSEW.docx
Lindsay ClaggettWeek 8 Discussion Post Back PainCOLLAPSEW.docxLindsay ClaggettWeek 8 Discussion Post Back PainCOLLAPSEW.docx
Lindsay ClaggettWeek 8 Discussion Post Back PainCOLLAPSEW.docx
 

More from Satoshi Kajiyama

Biomechanics of the thorax research evidence and clinical expertise
Biomechanics of the thorax   research evidence and clinical expertiseBiomechanics of the thorax   research evidence and clinical expertise
Biomechanics of the thorax research evidence and clinical expertiseSatoshi Kajiyama
 
Chornic pain and the thracic spine
Chornic pain and the thracic spineChornic pain and the thracic spine
Chornic pain and the thracic spineSatoshi Kajiyama
 
Regional interdependence and manual therapy directed at the thoracic spine
Regional interdependence and manual therapy directed at the thoracic spineRegional interdependence and manual therapy directed at the thoracic spine
Regional interdependence and manual therapy directed at the thoracic spineSatoshi Kajiyama
 
Clin infect dis. 2014-stevens-cid ciu296
Clin infect dis. 2014-stevens-cid ciu296Clin infect dis. 2014-stevens-cid ciu296
Clin infect dis. 2014-stevens-cid ciu296Satoshi Kajiyama
 
Br j sports med 2014 effectiveness of exercise interventions
Br j sports med 2014 effectiveness of exercise interventionsBr j sports med 2014 effectiveness of exercise interventions
Br j sports med 2014 effectiveness of exercise interventionsSatoshi Kajiyama
 
Br j sports med 2013-scott-536-44
Br j sports med 2013-scott-536-44Br j sports med 2013-scott-536-44
Br j sports med 2013-scott-536-44Satoshi Kajiyama
 
Basics of soft tissue examination
Basics of soft tissue examinationBasics of soft tissue examination
Basics of soft tissue examinationSatoshi Kajiyama
 
A neuroscience approach_to_managing_athletes_with_low_back_pain_puentedura_ph...
A neuroscience approach_to_managing_athletes_with_low_back_pain_puentedura_ph...A neuroscience approach_to_managing_athletes_with_low_back_pain_puentedura_ph...
A neuroscience approach_to_managing_athletes_with_low_back_pain_puentedura_ph...Satoshi Kajiyama
 
Proteinuria treatment algorithm
Proteinuria treatment algorithmProteinuria treatment algorithm
Proteinuria treatment algorithmSatoshi Kajiyama
 
Treatment of sacroiliac_joint_dysfunction_nata
Treatment of sacroiliac_joint_dysfunction_nataTreatment of sacroiliac_joint_dysfunction_nata
Treatment of sacroiliac_joint_dysfunction_nataSatoshi Kajiyama
 
Stride length for mlb pitchers march 2014
Stride length for mlb pitchers march 2014Stride length for mlb pitchers march 2014
Stride length for mlb pitchers march 2014Satoshi Kajiyama
 
Sports health overhead athlete part 1 and 2
Sports health overhead athlete part 1 and 2Sports health overhead athlete part 1 and 2
Sports health overhead athlete part 1 and 2Satoshi Kajiyama
 

More from Satoshi Kajiyama (20)

Biomechanics of the thorax research evidence and clinical expertise
Biomechanics of the thorax   research evidence and clinical expertiseBiomechanics of the thorax   research evidence and clinical expertise
Biomechanics of the thorax research evidence and clinical expertise
 
Prp and hamstring injury
Prp and hamstring injuryPrp and hamstring injury
Prp and hamstring injury
 
Chornic pain and the thracic spine
Chornic pain and the thracic spineChornic pain and the thracic spine
Chornic pain and the thracic spine
 
Regional interdependence and manual therapy directed at the thoracic spine
Regional interdependence and manual therapy directed at the thoracic spineRegional interdependence and manual therapy directed at the thoracic spine
Regional interdependence and manual therapy directed at the thoracic spine
 
Hs rehab 2
Hs rehab 2Hs rehab 2
Hs rehab 2
 
Hs rehab
Hs rehabHs rehab
Hs rehab
 
Clin infect dis. 2014-stevens-cid ciu296
Clin infect dis. 2014-stevens-cid ciu296Clin infect dis. 2014-stevens-cid ciu296
Clin infect dis. 2014-stevens-cid ciu296
 
Br j sports med 2014 effectiveness of exercise interventions
Br j sports med 2014 effectiveness of exercise interventionsBr j sports med 2014 effectiveness of exercise interventions
Br j sports med 2014 effectiveness of exercise interventions
 
Br j sports med 2013-scott-536-44
Br j sports med 2013-scott-536-44Br j sports med 2013-scott-536-44
Br j sports med 2013-scott-536-44
 
Basics of soft tissue examination
Basics of soft tissue examinationBasics of soft tissue examination
Basics of soft tissue examination
 
Baseball ue injuries
Baseball ue injuriesBaseball ue injuries
Baseball ue injuries
 
Athletic pubalgia mgmt
Athletic pubalgia mgmtAthletic pubalgia mgmt
Athletic pubalgia mgmt
 
Assorted handouts #2
Assorted handouts #2Assorted handouts #2
Assorted handouts #2
 
A neuroscience approach_to_managing_athletes_with_low_back_pain_puentedura_ph...
A neuroscience approach_to_managing_athletes_with_low_back_pain_puentedura_ph...A neuroscience approach_to_managing_athletes_with_low_back_pain_puentedura_ph...
A neuroscience approach_to_managing_athletes_with_low_back_pain_puentedura_ph...
 
Proteinuria treatment algorithm
Proteinuria treatment algorithmProteinuria treatment algorithm
Proteinuria treatment algorithm
 
Treatment of sacroiliac_joint_dysfunction_nata
Treatment of sacroiliac_joint_dysfunction_nataTreatment of sacroiliac_joint_dysfunction_nata
Treatment of sacroiliac_joint_dysfunction_nata
 
Stride length for mlb pitchers march 2014
Stride length for mlb pitchers march 2014Stride length for mlb pitchers march 2014
Stride length for mlb pitchers march 2014
 
Tendon anatomy diagram
Tendon anatomy diagramTendon anatomy diagram
Tendon anatomy diagram
 
Sports health overhead athlete part 1 and 2
Sports health overhead athlete part 1 and 2Sports health overhead athlete part 1 and 2
Sports health overhead athlete part 1 and 2
 
Sleep article
Sleep articleSleep article
Sleep article
 

Jospt.2014 abd aneurysm

  • 1. 500 | july 2014 | volume 44 | number 7 | journal of orthopaedic & sports physical therapy [ resident’s case problem ] L ow back pain is a common condition routinely managed by physical therapists and accounts for up to 53% of all client visits.1 It is also the fifth most common reason for all physician visits in the United States,19 with direct health care costs related to the management of low back pain in the United States estimated to be more than $85 billion annually.30 Unfortunately, despite increasing medi- cal expenditures, the prevalence of chron- ic, disabling low back pain continues to increase.30 Low back pain is most commonly caused by mechanical dysfunction, which can be defined as symptoms that are re- lated to the musculoskeletal system and vary with movement.40 However, serious nonmechanical spinal disorders (eg, neo- plasm, infection, inflammatory arthritis, fracture) or visceral disease (eg, gastroin- testinal, genitourinary, vascular) may be the cause of low back pain in a small per- centage of patients.22 Though mechanical dysfunction accounts for approximately 97% of low back pain cases, nonmechani- cal spinal disorders and visceral disease account for approximately 1% and 2% of low back pain cases, respectively.22 One type of visceral disease that may cause low back pain is an abdominal aor- tic aneurysm (AAA), which is an abnor- mal dilation in a weakened or diseased arterial wall.15 By convention, an infra- renal aorta that is 3 cm in diameter or larger is considered aneurysmal.12,26 The diameter of the infrarenal aorta is the strongest known predictor of risk for rup- ture,2 with the risk substantially increas- ing with infrarenal aorta diameters of 5.0 cm or greater.14,18 The etiology of an AAA is not com- pletely understood, but atherosclerosis, degeneration, and chronic inflammation have been identified as contributing to its development.39 Major risk factors for an AAA include hypertension, an age of greater than 60 years, male sex, past his- tory of smoking, atherosclerosis, coronary artery disease, family history of AAA, and use of statins (TABLE 1).2,7,12,13,26 Factors that are negatively associated with AAA include female sex, black race, and diabetes.2,7,13,24,25 The purpose of this resident’s case problem was (1) to describe the clini- cal reasoning and decision making in a patient referred with a diagnosis of me- chanical low back pain who had an AAA TTSTUDY DESIGN: Resident’s case problem. TTBACKGROUND: The purpose of this report was to describe (1) the clinical reasoning that led a clinician to identify an abdominal aortic aneurysm (AAA) in a patient with low back pain requiring immediate medical referral, and (2) an evidence- based approach to clinical evaluation of patients with suspected AAA. TTDIAGNOSIS: The patient was unable to identify a specific mechanism of injury for his low back pain, lacked aggravating/easing factors for his symptoms, and complained of night pain and an inability to ease his symptoms with position changes. While the patient’s symptoms remained unchanged during physical examination of the lum- bar spine and hip, abdominal palpation revealed a strong, nontender pulsation over the midline of the upper and lower abdominal quadrants. Due to concern for an AAA, the patient was immediately referred to his physician. Subsequent computed tomography imaging revealed a prominent AAA, which measured up to 5.5 cm in greatest dimen- sion and extended from below the renal arteries to the bifurcation of the iliac arteries. The patient ini- tially deferred surgical intervention but eventually consented 6 months later, after repeat computed tomography imaging revealed that the AAA had progressed to 6.7 cm in greatest dimension. TTDISCUSSION: It is essential for physical therapists to be familiar with a diagnostic pathway to help identify AAA in patients presenting with apparent musculoskeletal complaints. Knowledge of the risk factors for AAA, understanding how to screen for nonmusculoskeletal symptoms, and a basic competence in abdominal palpation and how to interpret findings will help with the clinician’s clinical decision making. TTLEVEL OF EVIDENCE: Differential diagnosis, level 4. J Orthop Sports Phys Ther 2014;44(7):500- 507. Epub 25 April 2014. doi:10.2519/ jospt.2014.4935 TTKEY WORDS: abdomen, aorta, clinical reasoning, palpation 1 Department of Physical Therapy, Charleston Air Force Base, Charleston, SC. 2 Physical Therapy Department, Daemen College, Amherst, NY. 3 Department of Physical Therapy, Fort McNair, Washington, DC. The opinions or assertions contained herein are the private views of the authors and are not to be construed as official or as reflecting the views of the US Air Force or the Department of Defense. The authors certify that they have no affiliations with or financial involvement in any organization or entity with a direct financial interest in the subject matter or materials discussed in the article. Address correspondence to Dr Joshua J. Van Wyngaarden, 1311 Hitchcock Avenue, Charleston AFB, SC 29404. E-mail: josh.vanwyngaarden@gmail.com t Copyright ©2014 Journal of Orthopaedic & Sports Physical Therapy® JOSHUA J. VAN WYNGAARDEN, PT, DPT1 • MICHAEL D. ROSS, PT, DHSc2 • BENJAMIN R. HANDO, PT, DSc3 Abdominal Aortic Aneurysm in a Patient With Low Back Pain 44-07 Van Wyngaarden.indd 500 6/17/2014 7:37:25 PM JournalofOrthopaedic&SportsPhysicalTherapy® Downloadedfromwww.jospt.orgatonAugust12,2014.Forpersonaluseonly.Nootheruseswithoutpermission. Copyright©2014JournalofOrthopaedic&SportsPhysicalTherapy®.Allrightsreserved.
  • 2. journal of orthopaedic & sports physical therapy | volume 44 | number 7 | july 2014 | 501 requiring immediate medical referral, and (2) to describe an evidence-based ap- proach to clinical evaluation of patients with suspected AAA. DIAGNOSIS Chief Complaint T he patient was a 58-year-old Caucasian man referred to a physi- cal therapist with a primary com- plaint of low back pain and a secondary complaint of abdominal pain. The pa- tient was employed as a government contractor, and his job consisted primar- ily of desk work. The patient performed 30 minutes of cardiovascular exercise (elliptical machine) 7 days per week and weight training 6 days per week. He had continued this exercise schedule despite his low back and abdominal pain. Symptom Location, Description, and Behavior The patient’s low back pain was located centrally at the L3 through L5 spinal levels. He characterized the pain as a constant, deep, dull ache that varied in intensity, with intermittent throbbing sensations (FIGURE 1). During the physical therapist’s initial encounter, the patient rated his current resting pain as 3 on a verbal numeric pain rating scale ranging from 0 to 10, with 0 as no pain and 10 as the worst pain imaginable. Using the same pain rating scale, the patient rated his pain since the pain first presented as 3 at its lowest level and 7 at its typical maximum level. The patient also reported abdominal pain that was centrally located, start- ing in the epigastric region and extend- ing inferiorly to the superior margin of the hypogastric region. He described his abdominal pain as a bloated feeling that progressed to a sharp, intense stabbing pain when aggravated. He currently rated his abdominal pain as 1; he also stated that 1 was the minimum pain he could feel, and the typical maximum pain was 4. The patient expressed that the low back pain and abdominal pain seemed to be related and increased proportionally; that is, when the low back pain increased, the abdominal pain increased as well. Symptom Behavior The patient was unable to describe any specific aggravating factors for his low back or abdominal pain. More specifical- ly, he denied any difficulty or increase in symptoms with aerobic exercise, climb- ing stairs, lumbar flexion/extension ac- tivities, or resistance training. He also denied an increase in symptoms with coughing, sneezing, and taking a deep breath. Taking naproxen was the only easing factor the patient identified, and it would reportedly lessen both his low back and abdominal pain temporarily. He had difficulty falling asleep at night, because the pain in his lower back would increase and be followed by an increase in abdominal pain. Once he fell asleep, he would typically wake at least once each night because of pain in his low back and abdomen. Position changes did not increase or decrease his symptoms. TABLE 1 Risk Factors Associated With Abdominal Aortic Aneurysm and Their Corresponding Odds Ratios7,13 Risk Factor Odds Ratio* Age 1.71 (1.61, 1.82) increase for every 7-y increase Male sex† 2.66 (1.69, 4.20) Hypertension† 1.44 (1.27, 1.63) Smoking history Past history† 1.5-2.4 (0.54, 6.75) Current smoker 6.19-13.72 (2.86, 30.78) Atherosclerosis† Not reported in literature Family history 1.94 (1.63, 2.32) Coronary artery disease 1.52 (1.37, 1.68) Use of statins 3.77 (1.45, 9.81) *Values in parentheses are 95% confidence interval. † Indicates the risk factors present in this patient. FIGURE 1. Location and description of patient’s symptoms at the time of the initial physical therapy visit. Checkmarks denote symptom-free areas. 44-07 Van Wyngaarden.indd 501 6/17/2014 7:37:25 PM JournalofOrthopaedic&SportsPhysicalTherapy® Downloadedfromwww.jospt.orgatonAugust12,2014.Forpersonaluseonly.Nootheruseswithoutpermission. Copyright©2014JournalofOrthopaedic&SportsPhysicalTherapy®.Allrightsreserved.
  • 3. 502 | july 2014 | volume 44 | number 7 | journal of orthopaedic & sports physical therapy [ resident’s case problem ] To resume sleeping, he would typically have to lie in bed and wait 45 minutes to 1 hour until he fell asleep, or take one 500-mg naproxen. History of Current Condition The patient reported that his pain began 8 days prior to his initial evaluation by the physical therapist. He denied a spe- cific mechanism of injury or trauma that preceded his symptoms. His symptoms came on suddenly while sitting in his ho- tel room when he was away on a business trip. He described his initial symptoms as a “sudden, severe pain” in his lower back and abdomen that forced him to lie su- pine for 30 minutes. Over the course of the next 8 days, the pain in his low back and abdomen slowly improved. The pa- tient denied a personal past history of low back or abdominal pain prior to this incident. One day prior to being seen by the physical therapist, the patient was evalu- ated by his primary care physician and diagnosed with mechanical low back pain and abdominal pain that was related to the low back pain, both of which were deemed musculoskeletal in nature. Prior to referral to the physical therapist, plain- film radiographs were completed, which revealed mild degenerative changes of the L3 through S1 spinal levels. The phy- sician subsequently prescribed naproxen (500 mg), to be taken twice daily for the next 10 to 14 days, and a physical therapy referral. Past Medical History The patient had a history of hyperten- sion and atherosclerosis, for which he was currently taking amlodipine (5 mg), vytorin (10 mg), hyzaar (12.5 mg), and aspirin (81 mg) once daily. He indicated that he had smoked for 2 years in the early 1970s, amounting to 1 pack-year total, but has not used tobacco products since. He denied numbness and tingling in his upper and lower extremities, recent infection, fatigue, weight changes, fever, malaise, saddle anesthesia, pulsation in his abdomen, and pain with the Valsalva maneuver or coughing. Heartburn, in- digestion, changes in appetite, difficulty swallowing, and changes in bowel and bladder habit were also all denied. He denied a family history of hypertension, diabetes mellitus, cancer, coronary artery disease, and AAA. Clinical Impression Post–History Intake The patient had a primary complaint of low back pain and a secondary complaint of abdominal pain that appeared to be re- lated. When considering the differential diagnosis, initial hypotheses were con- sidered in 3 categories: mechanical (eg, musculoskeletal condition), nonmechan- ical (eg, neoplasm, infection, inflamma- tory arthritis, fracture), and visceral (eg, gastrointestinal, genitourinary, or vascu- lar). This approach to differential diag- nosis allowed the clinician to determine whether the patient’s condition was ap- propriate for physical therapist manage- ment or an additional physician consult was required.3 Mechanical dysfunction accounts for approximately 97% of low back pain cas- es, suggesting a high probability that the vast majority of patients with low back pain will have pain that is mechanical in nature. Lumbar dysfunction may also cause abdominal pain.10,23 From the pa- tient history, however, the physical ther- apist identified components suggesting that the patient’s symptoms might not be mechanical in origin, such as the patient’s inability to identify a specific mechanism of injury to the sudden onset of symp- toms, the inability to identify any aggra- vating or easing factors for his symptoms, and night pain with an inability to ease his symptoms with changes in position. Additionally, Sparkes et al37 developed 2 specific clusters of questions to be used in identifying patients with abdominal pain that is musculoskeletal in origin (TABLE 2). These question clusters have high specificity (96%), suggesting that this tool is most useful for ruling in a muscu- loskeletal cause of abdominal pain. The patient described in this report could not answer “yes” to either of the first 2 ques- tions of cluster 1, suggesting that he was negative for this question cluster, which decreased the likelihood for a musculo- skeletal origin of the patient’s abdominal symptoms.37 The sensitivity (0.67) and negative likelihood ratio (LR) (0.39) as- sociated with the cluster are relatively weak, suggesting that a musculoskeletal cause of the patient’s symptoms was still high. However, when considered in the context of the other historical findings, the fact that cluster 1 was negative gave the clinician added suspicion that the symptoms could be related to nonmus- culoskeletal pathology. There were also components of the TABLE 2 Musculoskeletal Abdominal Pain Questionnaire37 Cluster 1* 1. Does coughing, sneezing, or taking a deep breath make your pain feel worse? (yes) 2. Do activities such as bending, sitting, lifting, twisting, or turning over in bed make your pain feel worse? (yes) 3. Has there been any change in your bowel habit since the start of your symptoms? (no) Cluster 2† 1. Does eating certain foods make your pain feel worse? (no) 2. Has your weight changed since your symptoms started? (no) *A patient answering “yes” to either of the first 2 and “no” to the third question of cluster 1 suggests a high probability that abdominal pain is musculoskeletal in origin (sensitivity, 0.67; specificity, 0.84; positive likelihood ratio, 4.2; negative likelihood ratio, 0.39). † If the patient answers “no” to both questions in cluster 2, in addition to corresponding responses to the questions in cluster 1, there is an even greater probability that abdominal pain is musculoskeletal in origin (sensitivity, 0.67; specificity, 0.96; positive likelihood ratio, 16.8; negative likelihood ratio, 0.34). 44-07 Van Wyngaarden.indd 502 6/17/2014 7:37:26 PM JournalofOrthopaedicSportsPhysicalTherapy® Downloadedfromwww.jospt.orgatonAugust12,2014.Forpersonaluseonly.Nootheruseswithoutpermission. Copyright©2014JournalofOrthopaedicSportsPhysicalTherapy®.Allrightsreserved.
  • 4. journal of orthopaedic sports physical therapy | volume 44 | number 7 | july 2014 | 503 patient history that the physical therapist identified and assisted in determining if the patient’s symptoms were nonmechan- ical in origin. The incidence of cancer as a cause of low back pain in a primary care setting is 0.66%, or approximately 6 to 7 cases for every 1000 patients with low back pain.8 Therefore, while the probabil- ity of cancer as a cause of low back pain is low, evidence-based screening strate- gies suggest that cancer may be ruled out with 100% sensitivity as a cause of low back pain if there is no previous his- tory of cancer, the patient is improving with conservative medical treatment, the patient is younger than 50 years of age, and there is no unexplained weight loss.8 These 4 clinical findings also had high positive LRs when assessing the accu- racy of clinical findings for detecting the presence of cancer resulting in low back pain.8 The patient described in this report had 3 of the 4 findings that allow cancer to be ruled out with 100% sensitivity. Although cancer could not be ruled out with 100% sensitivity, it is a rare cause of low back pain, and the patient did not demonstrate any of the strong pre- dictive findings for cancer as a cause of low back pain.8 In addition, Henschke et al20 conducted a Cochrane review to as- sess the diagnostic accuracy of red flags in screening for spinal malignancy in patients with low back pain. This review found a previous history of malignancy to be a clinically meaningful finding for detecting cancer. Given these factors, in addition to the relatively benign findings on plain-film radiographs, it was believed that cancer was an unlikely cause of low back pain. Given that the patient did not report a fever, fatigue, or malaise, it was believed that an infection was also un- likely.3,22,31,41 Additionally, given the lack of constitutional symptoms and no re- ports of morning stiffness, the likelihood of inflammatory arthritis as a cause of the patient’s low back and abdominal pain appeared to be low as well.17,22,31,34,35 Final- ly, it was highly unlikely that the patient had a spinal fracture. Vertebral fractures are the most common serious pathology associated with low back pain.21 Red-flag questions that are useful in identifying spinal fracture include significant trauma (positive LR, 10.03; negative LR, 0.77), older age (positive LR, 11.19; negative LR, 0.52), and prolonged corticosteroid use (positive LR, 48.50; negative LR, 0.75).21,42 Henschke et al21 created a diag- nostic rule to identify vertebral fracture using the following variables: female sex, age greater than 70 years, significant trauma (major in young patients, minor in elderly patients), and prolonged use of corticosteroids. When at least 1 of these variables was positive, the positive and negative LRs were 1.8 and 0.24, respec- tively.21 With 2 positive features, the posi- tive and negative LRs increased to 15.5 and 0.39, respectively; 3 or more positive variables yielded a positive LR of 218.3 and a negative LR of 0.62.21 In addition to the absence of evidence of spinal fracture on plain-film radiographs, none of these variables were present in this patient, further decreasing the likelihood that a vertebral fracture was causing low back pain. While mechanical dysfunction of the lumbar spine accounts for the vast major- ity of symptoms experienced by patients with low back pain, the majority of pa- tients with abdominal pain usually have a visceral cause, and in all patients with ab- dominal pain, visceral pathology should be ruled out.37 Furthermore, visceral dis- ease is a more frequent cause of low back pain compared to other nonmechanical spinal disorders.22 With regard to visceral disorders, initial screening questions for the gastrointestinal (eg, patient denied heartburn, nausea, indigestion, changes in appetite, difficulty swallowing, consti- pation, and changes in frequency of bowel movements) and genitourinary (patient denied pain with urination or changes in urinary frequency) systems were nega- tive.3 Although each of these findings does not necessarily rule out the presence of a gastrointestinal or genitourinary dis- order, the cluster of negative findings did suggest that the patient’s symptoms were most likely not related to these systems. Considering vascular pathology, an AAA is a rare cause of low back pain, and in many cases patients experience no symp- toms at all.27 However, the patient in this case indicated that he had 4 of the 8 risk factors associated with increased preva- lence of AAA (TABLE 1).2,7,12,13,26 Physical Examination The patient’s height was 188 cm, and he weighed 90.7 kg the day of the examina- tion (body mass index, 25.7 kg/m2 ). Vi- sual observation revealed a nonantalgic gait. A postural examination revealed a forward head and rounded shoulders, with a decreased lumbar lordosis. The iliac crests, anterior superior iliac spine, posterior superior iliac spine, and greater trochanters were level and in the same horizontal plane bilaterally. Lumbar active range of motion and associated symptom responses were as- sessed with the patient in the standing position. The low back and abdominal pain remained unchanged during active range-of-motion testing, and all motions were within normal limits. Because the patient’s symptoms remained unchanged with active range-of-motion assessment, overpressure at the end range of active range of motion was applied to lumbar flexion, extension, and bilateral sidebend- ing. The low back and abdominal pain also remained unchanged during over- pressure assessment for each movement. Seated hip active range of motion for flexion, internal rotation, and external ro- tation was within normal limits and pain free bilaterally. Passive range of motion with overpressure was then applied for hip flexion, internal rotation, and exter- nal rotation bilaterally, with the patient lying supine. Passive left hip flexion with overpressure slightly increased the pa- tient’s low back pain to 4 on the numeric pain rating scale, but the pain immedi- ately returned to the patient’s baseline pain level of 3 when his hip was taken out of the position of left hip flexion with overpressure. The patient did not note a change in his abdominal pain with hip range-of-motion assessment. 44-07 Van Wyngaarden.indd 503 6/17/2014 7:37:26 PM JournalofOrthopaedicSportsPhysicalTherapy® Downloadedfromwww.jospt.orgatonAugust12,2014.Forpersonaluseonly.Nootheruseswithoutpermission. Copyright©2014JournalofOrthopaedicSportsPhysicalTherapy®.Allrightsreserved.
  • 5. 504 | july 2014 | volume 44 | number 7 | journal of orthopaedic sports physical therapy [ resident’s case problem ] With the patient positioned prone, spring testing through posterior-to-an- terior pressures over the spinous pro- cesses of T10 through L4 suggested mild hypomobility but no increase in the pa- tient’s symptoms. However, the patient did report a slight increase in low back pain with spring testing at the L5 level, and there was hypomobility noted at this level as well. A sacral spring test was then performed, which did not change the pa- tient’s symptoms. The patient denied any change in his abdominal pain with spring testing from T10 through S1. The patient was then positioned su- pine, and an abdominal assessment was performed. Visual inspection of the pa- tient’s abdomen did not reveal any skin abnormalities, abdominal masses, or abnormal movement of the abdominal wall. Systematic palpation in each of the 4 abdominal quadrants did not provoke an increase in the patient’s symptoms. However, the clinician immediately no- ticed the presence of a very strong ab- dominal pulsation that lateralized from the midline bilaterally and was measured at approximately 5 cm in width. The pul- sation was palpable from the epigastric to the hypogastric regions. Following this finding, the clinician elected to im- mediately stop the objective examination and contacted the patient’s primary care provider to discuss the findings from the evaluation. Clinical Impression Post–Physical Examination The patient’s low back and abdominal pain was not reproduced with any of the physical examination tests and measures performed, with the exception of a mild, temporary increase in low back pain with overpressure into hip flexion and L5 palpation. In the majority of patients with nonspecific mechanical low back pain, pain is reproduced with lumbar range-of-motion testing and/or lumbar palpation.4,5 Therefore, the absence of symptom reproduction during the physi- cal examination further elevated concern for the presence of visceral pathology, which led to a thorough palpation of the abdomen. During abdominal palpation, a very strong, nontender abdominal pulsa- tion at least 5 cm in width was detected, which was concerning for AAA.3,31 Diagnosis After discussing the case, the referring physician immediately ordered an ultra- sound imaging study of the abdomen, which demonstrated dilation of the dis- tal abdominal aorta up to 5.5 cm wide (FIGURE 2). The physician subsequently ordered a computed tomography scan confirming the presence of a prominent AAA measuring 5.5 cm in greatest di- mension and an 80° angulation (FIGURE 3). The patient was immediately referred to a vascular surgeon. However, after a thorough discussion with the vascular surgeon, the patient decided to forgo immediate surgical intervention, believ- ing that the risks incurred from surgery outweighed the potential benefit. Given this decision, the patient was instructed to return biannually for repeat computed tomography scanning to determine if the AAA was progressing in size. The patient reported that his primary complaint of low back pain and secondary complaint of abdominal pain had completely sub- sided approximately 3 weeks after the initial onset of symptoms. Six months following the initial di- agnosis of the AAA, repeat computed tomography scanning revealed that the FIGURE 3. Initial computed tomography image, which demonstrated a prominent abdominal aortic aneurysm, which measured up to 5.6 cm in greatest dimension (line) and extended from below the renal arteries to the bifurcation of the iliac arteries. The aorta also buckled to the right, with approximately 80° of angulation present 1.3 cm distal to the left renal artery (arrow). FIGURE 2. Initial ultrasound image demonstrating a 5.5-cm dilation of the distal abdominal aorta (region between the plus signs). 44-07 Van Wyngaarden.indd 504 6/17/2014 7:37:27 PM JournalofOrthopaedicSportsPhysicalTherapy® Downloadedfromwww.jospt.orgatonAugust12,2014.Forpersonaluseonly.Nootheruseswithoutpermission. Copyright©2014JournalofOrthopaedicSportsPhysicalTherapy®.Allrightsreserved.
  • 6. journal of orthopaedic sports physical therapy | volume 44 | number 7 | july 2014 | 505 aneurysm had increased by 1.2 to 6.7 cm in greatest dimension and to 90° of angulation (FIGURE 4). Interestingly, the patient reported that he did not have any further episodes of low back or abdomi- nal pain since the symptoms resolved ap- proximately 5 months earlier, despite the aneurysm’s progression. The average rate of expansion of the typical AAA is from 0.3 to 0.4 cm per year.14,18,29 Patients who have AAA expansion rates of 0.5 cm in a 12-month time frame are considered can- didates for elective surgical repair.16 Due to the rapid progression of the AAA, the patient accepted the risks of an open ab- dominal repair. The patient requested an endovascular repair, but the increasing angulation of the aorta and the weaken- ing aortic wall made an open repair the only viable option. TABLE 3 provides rela- tive dates of the patient’s medical visits and procedures. Following surgery, the patient re- turned to walking 30 minutes to 1 hour daily. At 6 months following surgery, the patient was allowed to initiate a low- weight, high-repetition weight-training routine. At 9 months following surgery, the patient did not report any instances of low back or abdominal pain since the repair. DISCUSSION I dentifying individuals with AAA is a challenging task for any clinician. There are no established algorithms or clinical prediction rules to determine with reasonable certainty whether an AAA is present in a patient. Additionally, many AAAs are completely asymptom- atic, which further complicates the clini- cal picture.27 However, there is evidence in the peer-reviewed literature that can inform clinicians in their clinical reason- ing and thought process. Knowledge of the risk factors for AAA, systematic ap- plication of screening techniques for non- musculoskeletal symptoms, and a basic competence in abdominal palpation and how to interpret findings can help iden- tify AAA in a patient presenting with ap- parent musculoskeletal complaints. Understanding the risk factors for AAA can help identify patients who should be referred for further testing. The major risk factors and associated odds ratios for AAA are listed in TABLE 1. There is discrepancy in the literature as far as when patients should undergo medical screening for AAA.6,12,28 In obese male patients over the age of 60, Led- erle et al28 recommended ultrasound screening examinations regardless of the physical examination findings. Flem- ing et al12 conducted a systematic review that looked at the benefits and harms of population-based AAA screening. They concluded that screening will reduce age-related mortality in men between the ages of 65 and 75 years.12 According to Cosford and Leng,7 an age of greater than 65 years, past history of smok- ing, male sex, and family history are the most significant AAA risk factors (TABLE 1).7,12 Therefore, if a male patient in his sixth decade of life has 1 or more addi- tional risk factors for AAA, a referral to the primary care provider with a recom- mendation for an ultrasound screening examination may be indicated. In patients with complaint of abdomi- nal pain, the question cluster suggested by Sparkes et al37 can serve as a valuable clinical decision tool when determining whether a patient has symptoms of mus- culoskeletal origin (TABLE 2). Additional- ly, the characteristics of abdominal pain can provide insight into the structures responsible for symptoms. Abdominal pain of musculoskeletal origin may pre­ sent as a deep, sharp, localized, cramping and aching pain.32 In comparison, pain FIGURE 4. Computed tomography scan completed 6 months after the initial scan seen in FIGURE 3. This image demonstrated interval progression of the abdominal aortic aneurysm to 6.7 cm in greatest dimension (line). There was also significant angulation at the neck of the aneurysm, measuring at least 90° (arrow). TABLE 3 Relative Timing of the Patient’s Medical Visits or Procedures Date Medical Visits or Procedures Day 1 • Initial physical therapy visit for a primary complaint of low back pain and a secondary complaint of abdominal pain • Physical therapist informed the patient and the referring provider of the concern of an abdominal aortic aneurysm • Ultrasound imaging and computed tomography scan completed, which confirmed the presence of an abdominal aortic aneurysm • Initial vascular surgery visit Day 17 • Second vascular surgery visit Day 31 • Third vascular surgery visit • Decision made to forgo immediate surgical intervention of abdominal aortic aneurysm • Primary complaint of low back pain and secondary complaint of abdominal pain had completely subsided • Patient instructed to complete repeat computed tomography scan in 6 mo Day 204 • Fourth vascular surgery visit • Repeat computed tomography scan results evaluated, which demonstrated interval progression of abdominal aortic aneurysm Day 302 • Open vascular repair of abdominal aortic aneurysm 44-07 Van Wyngaarden.indd 505 6/17/2014 7:37:28 PM JournalofOrthopaedicSportsPhysicalTherapy® Downloadedfromwww.jospt.orgatonAugust12,2014.Forpersonaluseonly.Nootheruseswithoutpermission. Copyright©2014JournalofOrthopaedicSportsPhysicalTherapy®.Allrightsreserved.
  • 7. 506 | july 2014 | volume 44 | number 7 | journal of orthopaedic sports physical therapy [ resident’s case problem ] REFERENCES 1. Battie MC, Cherkin DC, Dunn R, Ciol MA, Wheel- er KJ. Managing low back pain: attitudes and treatment preferences of physical therapists. Phys Ther. 1994;74:219-226. 2. Blanchard JF, Armenian HK, Friesen PP. Risk factors for abdominal aortic aneurysm: re- sults of a case-control study. Am J Epidemiol. 2000;151:575-583. 3. Boissonnault WG. Primary Care for the Physical Therapist: Examination and Triage. 2nd ed. St Louis, MO: Elsevier/Saunders; 2011. 4. Boissonnault WG, Bass C. Pathological origins of trunk and neck pain: part I—pelvic and abdominal visceral disorders. J Orthop Sports Phys Ther. 1990;12:192-207. http://dx.doi. org/10.2519/jospt.1990.12.5.192 5. Boissonnault WG, Bass C. Pathological origins of trunk and neck pain: part II—disorders of the cardiovascular and pulmonary systems. J Or- thop Sports Phys Ther. 1990;12:208-215. http:// dx.doi.org/10.2519/jospt.1990.12.5.208 6. Chervu A, Clagett GP, Valentine RJ, Myers SI, Rossi PJ. Role of physical examination in detec- tion of abdominal aortic aneurysms. Surgery. 1995;117:454-457. 7. Cosford PA, Leng GC. Screening for ab- dominal aortic aneurysm. Cochrane Database Syst Rev. 2007:CD002945. http://dx.doi. org/10.1002/14651858.CD002945.pub2 8. Deyo RA, Diehl AK. Cancer as a cause of back pain: frequency, clinical presentation, and diagnostic strategies. J Gen Intern Med. 1988;3:230-238. 9. Edwards JZ, Weiner SD. Chronic back pain caused by an abdominal aortic aneurysm: case report and review of the literature. Orthopedics. 2003;26:191-192. 10. Feinstein B, Langton JN, Jameson RM, Schil- ler F. Experiments on pain referred from deep somatic tissues. J Bone Joint Surg Am. 1954;36-A:981-997. 11. Fink HA, Lederle FA, Roth CS, Bowles CA, Nelson DB, Haas MA. The accuracy of physical exami- nation to detect abdominal aortic aneurysm. Arch Intern Med. 2000;160:833-836. 12. Fleming C, Whitlock EP, Beil TL, Lederle FA. Screening for abdominal aortic aneurysm: a best-evidence systematic review for the U.S. Preventive Services Task Force. Ann Intern Med. 2005;142:203-211. 13. Forsdahl SH, Singh K, Solberg S, Jacob- sen BK. Risk factors for abdominal aortic aneurysms: a 7-year prospective study: the Tromsø Study, 1994-2001. Circulation. 2009;119:2202-2208. http://dx.doi.org/10.1161/ CIRCULATIONAHA.108.817619 14. Glimåker H, Holmberg L, Elvin A, et al. Natural history of patients with abdominal aortic aneu- rysm. Eur J Vasc Surg. 1991;5:125-130. 15. Goodman CC, Snyder TEK. Differential Diagnosis in Physical Therapy. 3rd ed. Philadelphia, PA: arising from visceral tissue is often de- scribed as dull, aching, cramping, throb- bing, burning, gnawing, or wave-like and is often poorly localized.32 Movement typically alters symptoms associated with musculoskeletal pain.4,5 In contrast, nonmusculoskeletal pain can be characterized as pain that cannot be provoked, alleviated, or eliminated by movement or position changes.4,5,15 If a patient is unable to identify specific ag- gravating and easing factors and the cli- nician is unable to reproduce symptoms with provocative testing, then a nonmus- culoskeletal origin of pain should be con- sidered and the case discussed with the referring physician.3,32 Abdominal palpation can play an im- portant role in identifying patients with AAA. However, the accuracy of palpation depends on the width of the aneurysm and the patient’s abdominal girth.11 Ab- dominal palpation has only a moderate overall ability in detecting smaller AAAs, but does appear very useful in identify- ing aneurysms large enough to warrant elective repairs.11,28,29 The sensitivity of palpation in patients with an abdominal girth less than 100 cm and an AAA 5.0 cm or greater is 100%.11 The patient in this case had a 95-cm waistline and had an AAA greater than 5 cm, resulting in a successful detection of the AAA during the palpation examination. Palpation of an AAA appears to be safe and has not been reported to precipitate rupture.27 If a palpable pulsation of 3.0 cm in width or greater is noticed, physician referral is indicated12 ; however, it’s important to re- member that aneurysms less than 5.0 cm are not as easily identifiable with palpa- tion. Therefore, sole reliance on abdomi- nal palpation is not recommended.11,24 It is important to note that the clini- cal examination plays a vital role in the recognition of AAAs, but should not be relied on to exclude their presence.29 If a referral to the primary care physi- cian for suspected AAA is indicated, the physician will likely order an abdominal ultrasound, as this can confirm the pres- ence or absence of an AAA (sensitivity, 0.97-1.0; specificity, 0.94-1.0; negative LR, 0-0.025; positive LR, 10.8 to infin- ity).28,33 Ultrasound screening is not only highly accurate in detecting AAAs33 but is also low cost and does not expose the patient to radiation.6,22,28 If the presence of an AAA is confirmed with ultrasound imaging, a computed tomography scan is the preferred imaging progression.16,38 While the sensitivity and specificity of computed tomography imaging of the ab- domen are lower than that of ultrasound imaging (sensitivity, 0.79-0.90; specific- ity, 0.77-0.91),36 it can detect additional aneurysms, evaluate the surrounding organs, and provide details on arterial- wall characteristics (dissection, rupture, or thickening).16 Computed tomogra- phy does have one major disadvantage: it exposes the patient to radiation.16,38 Magnetic resonance angiography yields similar results to those of computed to- mography without exposure to ionizing radiation, but high cost and limited avail- ability have prevented widespread adop- tion of this modality.16,38 CONCLUSION A lthough the majority of indi- viduals with AAA are asymp- tomatic at the time of diagnosis, some patients do present with a history and clinical presentation suggestive of AAA.9 This resident’s case problem de- scribed the decision making and clinical reasoning that led a clinician to cor- rectly identify the presence of an AAA in a patient referred from a primary care provider with a diagnosis of mechanical low back pain. Findings from the sub- jective interview and physical examina- tion led the clinician to suspect AAA as an explanation for the patient’s com- plaints. This suspicion was confirmed with a series of imaging studies, which led to expedited medical care and even- tual surgical intervention. t 44-07 Van Wyngaarden.indd 506 6/17/2014 7:37:29 PM JournalofOrthopaedicSportsPhysicalTherapy® Downloadedfromwww.jospt.orgatonAugust12,2014.Forpersonaluseonly.Nootheruseswithoutpermission. Copyright©2014JournalofOrthopaedicSportsPhysicalTherapy®.Allrightsreserved.
  • 8. journal of orthopaedic sports physical therapy | volume 44 | number 7 | july 2014 | 507 @ MORE INFORMATION WWW.JOSPT.ORG W.B. Saunders; 2000. 16. Gorski Y, Ricotta JJ. Weighing risks in abdominal aortic aneurysm. Best repaired in an elective, not an emergency, procedure. Postgrad Med. 1999;106:69-70, 75-80. 17. Gran JT. An epidemiological survey of the signs and symptoms of ankylosing spondylitis. Clin Rheumatol. 1985;4:161-169. 18. Guirguis EM, Barber GG. The natural history of abdominal aortic aneurysms. Am J Surg. 1991;162:481-483. 19. Hart LG, Deyo RA, Cherkin DC. Physician of- fice visits for low back pain. Frequency, clinical evaluation, and treatment patterns from a U.S. national survey. Spine (Phila Pa 1976). 1995;20:11-19. 20. Henschke N, Maher CG, Ostelo RW, de Vet HC, Macaskill P, Irwig L. Red flags to screen for malignancy in patients with low-back pain. Co- chrane Database Syst Rev. 2013;2:CD008686. http://dx.doi.org/10.1002/14651858.CD008686. pub2 21. Henschke N, Maher CG, Refshauge KM, et al. Prevalence of and screening for serious spinal pathology in patients presenting to primary care settings with acute low back pain. Arthritis Rheum. 2009;60:3072-3080. http://dx.doi. org/10.1002/art.24853 22. Jarvik JG, Deyo RA. Diagnostic evaluation of low back pain with emphasis on imaging. Ann Intern Med. 2002;137:586-597. http://dx.doi. org/10.7326/0003-4819-137-7-200210010-00010 23. Kellgren JH. Referred pains from muscle. Br Med J. 1938;1:325-327. 24. Lederle FA, Johnson GR, Wilson SE, et al. The aneurysm detection and management study screening program: validation cohort and final results. Aneurysm Detection and Management Veterans Affairs Cooperative Study Investigators. Arch Intern Med. 2000;160:1425-1430. 25. Lederle FA, Johnson GR, Wilson SE, et al. Preva- lence and associations of abdominal aortic an- eurysm detected through screening. Aneurysm Detection and Management (ADAM) Veterans Affairs Cooperative Study Group. Ann Intern Med. 1997;126:441-449. 26. Lederle FA, Johnson GR, Wilson SE, et al. Rela- tionship of age, gender, race, and body size to infrarenal aortic diameter. The Aneurysm Detec- tion and Management (ADAM) Veterans Affairs Cooperative Study Investigators. J Vasc Surg. 1997;26:595-601. 27. Lederle FA, Simel DL. The rational clinical exami- nation. Does this patient have abdominal aortic aneurysm? JAMA. 1999;281:77-82. http://dx.doi. org/10.1001/jama.281.1.77 28. Lederle FA, Walker JM, Reinke DB. Selective screening for abdominal aortic aneurysms with physical examination and ultrasound. Arch In- tern Med. 1988;148:1753-1756. 29. Lynch RM. Ruptured abdominal aortic aneu- rysm presenting as groin pain. Br J Gen Pract. 2002;52:320-321. 30. Martin BI, Deyo RA, Mirza SK, et al. Expendi- tures and health status among adults with back and neck problems. JAMA. 2008;299:656-664. http://dx.doi.org/10.1001/jama.299.6.656 31. Mechelli F, Preboski Z, Boissonnault WG. Dif- ferential diagnosis of a patient referred to physical therapy with low back pain: abdominal aortic aneurysm. J Orthop Sports Phys Ther. 2008;38:551-557. http://dx.doi.org/10.2519/ jospt.2008.2719 32. Rodeghero JR, Denninger TR, Ross MD. Abdomi- nal pain in physical therapy practice: 3 patient cases. J Orthop Sports Phys Ther. 2013;43:44- 53. http://dx.doi.org/10.2519/jospt.2013.4408 33. Rubano E, Mehta N, Caputo W, Paladino L, Sin- ert R. Systematic review: emergency department bedside ultrasonography for diagnosing sus- pected abdominal aortic aneurysm. Acad Emerg Med. 2013;20:128-138. http://dx.doi.org/10.1111/ acem.12080 34. Rudwaleit M, Metter A, Listing J, Sieper J, Braun J. Inflammatory back pain in ankylosing spon- dylitis: a reassessment of the clinical history for application as classification and diagnostic cri- teria. Arthritis Rheum. 2006;54:569-578. http:// dx.doi.org/10.1002/art.21619 35. Sieper J, van der Heijde D, Landewé R, et al. New criteria for inflammatory back pain in patients with chronic back pain: a real patient exercise by experts from the Assessment of SpondyloArthritis international Society (ASAS). Ann Rheum Dis. 2009;68:784-788. http://dx.doi. org/10.1136/ard.2008.101501 36. Silverstein MD, Pitts SR, Chaikof EL, Ballard DJ. Abdominal aortic aneurysm (AAA): cost- effectiveness of screening, surveillance of intermediate-sized AAA, and management of symptomatic AAA. Proc (Bayl Univ Med Cent). 2005;18:345-367. 37. Sparkes V, Prevost AT, Hunter JO. Derivation and identification of questions that act as predictors of abdominal pain of musculo- skeletal origin. Eur J Gastroenterol Hepatol. 2003;15:1021-1027. http://dx.doi.org/10.1097/01. meg.0000059173.46867.0c 38. Tang T, Boyle JR, Dixon AK, Varty K. Inflamma- tory abdominal aortic aneurysms. Eur J Vasc Endovasc Surg. 2005;29:353-362. http://dx.doi. org/10.1016/j.ejvs.2004.12.009 39. Thompson RW. Basic science of abdominal aor- tic aneurysms: emerging therapeutic strategies for an unresolved clinical problem. Curr Opin Cardiol. 1996;11:504-518. 40. Waddell G. The Back Pain Revolution. New York, NY: Churchill Livingstone; 1998. 41. Waldvogel FA, Papageorgiou PS. Osteo- myelitis: the past decade. N Engl J Med. 1980;303:360-370. http://dx.doi.org/10.1056/ NEJM198008143030703 42. Williams CM, Henschke N, Maher CG, et al. Red flags to screen for vertebral fracture in patients presenting with low-back pain. Cochrane Data- base Syst Rev. 2013;1:CD008643. http://dx.doi. org/10.1002/14651858.CD008643.pub2 NOTIFY JOSPT of Changes in Address Please remember to let JOSPT know about changes in your mailing address. The US Postal Service typically will not forward second-class periodical mail. Journals are destroyed, and the USPS charges JOSPT for sending them to the wrong address. You may change your address online at www.jospt.org. Visit Info Center for Readers, click Change of Address, and complete the online form. We appreciate your assistance in keeping JOSPT’s mailing list up to date. 44-07 Van Wyngaarden.indd 507 6/17/2014 7:37:30 PM JournalofOrthopaedicSportsPhysicalTherapy® Downloadedfromwww.jospt.orgatonAugust12,2014.Forpersonaluseonly.Nootheruseswithoutpermission. Copyright©2014JournalofOrthopaedicSportsPhysicalTherapy®.Allrightsreserved.